Did the test collect any data on the demographics of the takers, so that we can check whether the drop is within-group, or whether it’s just artifact of changing composition? For all we know, this might be just a case of Simpson paradox, where each subgroup actually improved, but the overall score distribution shifted downwards due to school children population growing more and more immigrant, after years of mass immigration. While US has excellent education, and immigrant children in America do better than their counterparts in their countries of origin, they don’t do as well as the modal group of white Americans, so as the composition changes, and the whites become less of a statistically dominating factor, the scores are expected to go down, even as the education quality improves.
Thank you for the high quality comment. I'll try to add some insight here for race. Using the reports [1] and [2], the difference for Grade 4 students are:
American Indian / Alaska Native - 2.5% in 2019 - 1.6% in 2023 - 515 in 2019 - 504 in 2023
Asian - 5.3% in 2019 - 4.3% in 2023 - 586 in 2019 - 571 in 2023
Black - 13.2% in 2019 - 15.5% in 2023 - 494 in 2019 - 468 in 2023
Hispanic - 25.8% in 2019 - 26.3% in 2023 - 508 in 2019 - 491 in 2023
Native Hawaiian / Other Pacific Islander - 1.7% in 2019 - 0.9% in 2023 - 500 in 2019 - 457 in 2023
Two or more races - 5.6% in 2019 - 8.1% in 2023 - 554 in 2019 - 542 in 2023
White - 45.9% in 2019 - 43.2% in 2023 - 559 in 2019 - 543 in 2023
It looks like all groups suffered at least 10 points in loss but these effects are definitely exaggerated by the reweighing of population proportions.
Thank you for looking it up! My personal hunch is that COVID related disruption in schooling is responsible for most of the within-group drop, and the rest is mostly a result of changes in educational policy, where many places deemphasize objective measures and standards, causing students to care less for these on the margin.
Personally, I suspect there's likely a generational non-COVID caused reduction in child welfare at play, the leading edge of which happened to get hidden by COVID.
The combo of more and more parents who work all the time and/or have delegated parenting to YouTube and mobile devices, plus charter schools (1) moving a chunk of non-IEP kids into more non-secular schools with lower standards and (2) concentrating behavior issue and IEP kids in public schools are basically rotting things from the inside out.
Would you care to share more about what this means?
"moving a chunk of non-IEP kids into more non-secular schools with lower standards"
When we applied to public schools for my child, the one we were most excited about was a charter school, not one run by the local school district.
As far as I can tell, charter schools have some selection pressure that is absent from government-run schools: if a charter school does poorly, parents don't choose it, and it closes. If a district-run school does poorly, the district will allocate kids there anyway.
In my area charter schools game the system to not deal with special needs kids and make them leave the charters. Many basically avoid doing evaluations and IEPs entirely so they can force them to go back to public schools.
The highest achieving charters have also been able to get mediocre non IEP kids to transfer back by setting them up to have trouble keeping up and not supporting them when they struggle.
Edit: And on the lower-performing side, you have schools focused on religion and/or patriotism, which tend to be less concerned about getting low performers out, but which aren't based as much around effective academics so much as ideology, and which do not meet the civic bar for education. Though I've also seen some that are just there to basically act like the charter equivalent of for-profit colleges: give parents in low SES areas a message of hope and then just minimize costs in the classroom.
> on the lower-performing side, you have schools focused on religion
That is an interesting contrast with the British equivalent. Religious schools are generally academically much better, so much so that people lie about their religion (and start turning up to church etc.) to get in (IIRC it is a condition of state funding that a minimum proportion of students are followers of the religion).
My experience in the US has been similar - most religious schools had higher performance than most public schools. Even the grading systems reflected the higher standards by using a 7 point scale rather than a 10 point scale. None of the religious schools I heard of had the "no zero" policies like some of the public schools where you get an automatic minimum 50% score just for writing your name on the test.
I'm sure there are outliers in either school type, but categorizing religious schools as lower performing doesn't to fit my experiences at all.
Yes. Catholic schools, for example, have long been known for far surpassing public school performance, so much so that non-Catholics send their children to Catholic schools. Apart from the better quality of the curricula, Catholic schools have never experienced school shootings, student relations are more genial and civilized, and you can be more confident that your child will be spared the unhinged ideologies du jour.
On top of that, they are quite generous and inexpensive as non-state schools. Listed tuition does vary considerably, as some elite Catholic schools are truly very expensive. However, they're not representative of typical tuition costs, and in either case, they usually offer financial assistance. They even lower tuition for each subsequent child you send to their school. Some schools (like Regis in NYC) are completely free, if you get in.
On top of that, Catholic laity have been founding schools independently in the last ~20 years or so (that is, these are private schools that are not under the direct authority of the bishop of the local diocese). These tend to emphasize a return to classical curricula (like the trivium and quadrivium), updated and supplemented accordingly.
(Those with a prejudice against religious schools as a whole also fail to understand that a guiding worldview is always present in any school. This cannot be avoided. In fact, it is nonsensical. It is the backdrop that determines the organization and structure of curricula in the first place. State schools in the modern liberal state naturally insinuate and teach a liberal worldview, one that bona fide Catholics merely tolerate as a matter of practicality, but utterly reject as a matter of principle, as the liberal worldview along with its liberal anthropology is at odds with the Catholic worldview. Education is ultimately a matter of intellectual formation which is something that entails some measure of moral formation as well. The liberal arts are first and foremost about freeing a person to be able to reason and reason proficiently, not about producing economic actors first and foremost, though you could naturally expect someone with this formation to be well-prepared for the world of work as well as a consequence of having been formed intellectually and morally.)
It depends on whether the focus is on spreading knowledge or preserving doctrine. Most religious schools I know are solidly in the former category, but I suspect there are exceptions.
This seems like a false dichotomy to me. They can do both. We could look at SAT scores, which show religious schools having significantly higher scores than public schools. So clearly they are learning the material.
In what now feels like a past life I knew a guy who had a perfect SAT score, I met him while visiting a family member who was his roommate at Patrick Henry College in VA.
He believed radio carbon dating was a lie and the earth was literally ~5k years old.
There are confounding factors that SAT scores don't surface.
That's why I'm saying one of the post above seems like a false dichotomy as the schools could do both. If the measures of school quality are test results, higher education, etc, then the religious schools would still exceed the public schools in those scores, which are the only measures we have.
An interesting question is, would that person in your example answer a question about radio carbon dating correctly on a test? It's possible they know the correct answer to mark but have separate philosophical beliefs. This would further support the idea that the religious schools have high academic achievement even if the beliefs differ.
My example was an anecdote, I would assume any one person with a perfect SAT score is an outlier across educational methodologies.
Self directed learning, reading comprehension, maths skills, critical thinking skills, ability to identify and reject sophistry, these abilities have some overlap perhaps but can each be skilled up separately and impact how a person can later function in roles that depend on the ability to identify true/false propositions.
Do you view the utility and practice of having overlapping confirmations and methodologies for radiometric dating (radiocarbon, potassium–argon, uranium–lead, etc.) to be a philosophic belief?
Do you think that it is wise to spend time and money formally training "high potential" people to have such deeply systemic errors in reasoning to the point that they flat reject peer reviewed, high confidence data and methods as a positive?
We aren't talking about being a Theist, Deist, ETC. We aren't talking about religious beliefs as most christians support radiometric dating, we are talking about educational materials for the young that reject following the data in pragmatic, well documented, well confirmed, high utility fields and rejects them out of pocket with untestable unobservable and sometimes demonstrably false alternatives. I don't want my curriculum to be the blind (or the sophists) leading the blind and to pretend these are equivalent because this self selected group trends towards higher SAT scores seems to miss the point.
I am pro-homeschooling and private schools generally and have multi generational exposure and participation in these things. Let a family believe whatever it wants to believe intellectually, I generally believe these variations to be positive in ensuring the human race is more diverse in exploring the "search space".
That being said, one of the smartest people I know, with all the raw materials in the world became a PHD scientist in molecular biology and bioinformatics and got nerd sniped by his YEC background, the years he's spent spinning his wheels due to an inability to follow the conclusions of his cognitive dissonance means he might as well not have become a scientist at all. It's enough that his family did that to him and it's fine or whatever, but we shouldn't have tax paid educators and curriculums participate in the cognitive crippling of children if our goal is to have a functional and productive society.
There are many people who hold beliefs that run contrary to the evidence on a variety of subjects from all educational backgrounds.
"...because this self selected group trends towards higher SAT scores seems to miss the point."
If it does, it's a point that hasn't been made yet. This entire post is about test scores, which is the way education is measured. If you're contending that public schools produce better reasoning than religious schools, then please provide that data.
"but we shouldn't have tax paid educators and curriculums participate in the cognitive crippling of children if our goal is to have a functional and productive society."
This part left me unclear, as this is also the same sort of complaint many parents are having about some of the ideologies being taught (or how they're being taught) in public schools, which are publicly funded. Most states do not publicly fund religious/private schools.
"Do you think that it is wise to spend time and money formally training "high potential" people to have such deeply systemic errors in reasoning to the point that they flat reject peer reviewed, high confidence data and methods as a positive?"
So how did your example achieve a PhD if he contradicted the peer reviewed evidence? I assume the review board and degree was from a secular school and you have implied the secular school process doesn't permit these sorts of reasoning flaws. Perhaps their reasoning on most subjects is solids and they have a few blind spots. This is generally true of most people, including those from public schools. Also, let it be known that many successful scientists have challenged existing positions successfully to discover new things. Science encourages challenging existing positions as part of the process to make new discoveries. This is one of the reasons there are so few "laws" in science and so many "theories".
"We aren't talking about religious beliefs as most christians support radiometric dating, we are talking about educational materials for the young that reject following the data in pragmatic, well documented, well confirmed, high utility fields and rejects them out of pocket with untestable unobservable and sometimes demonstrably false alternatives."
Ok... so what is your point? The vast majority of religious schools support radio cabon dating just as you admit that most religions do. I'm pretty confident that my comment chain has been using qualifiers to indicate that there may be outliers, but that in general religious schools do not have lower academic standards as indicated by the test score data. There may be outliers in any of the various school types, but the data is pretty clear that the current measures of academic success are test scores and post-secondary education, for which religious school score at least as well on. This refutes the original comment that religious schools have academically lower standards. Unless someone has actual data and not just anecdotal that says otherwise.
Outliers, bias due to self selection, bad/factually false curriculums we can observe to be in use (not simply worries pertaining to observed outcomes in an individual), institutional methods to force out special needs children to improve averages. These are all confounding factors to a simple and frankly wrong narrative. Perhaps the real issue is our universally appalling teaching in statistics.
Unless you have well designed co-hort studies most of the data just isn't really useful. The SAT, like any other measure that becomes the standard, is going to be gamed. I'm trying to highlight why a shallow and cursory glance at any data can be misleading and adding specific observations as food for thought.
Take it as you will. Test scores surely are directional and with universal declines that is worrying, but that isn't the sole topic of discussion in the thread is it?
The intial ellipses is pretty indicative, it seems clear that radiometric dating is a charity belief and is not something one thinks is a valid mechanism for acquiring accurate data and facts that map to observable, testable reality.
I think most people wouldn't have so much difficulty distinguishing philosophic positions and fact acquiring methodologies, I mean some people view physical reality as not existing and all material endeavor and observation to be a waste, surely that's an equally valid worldview to inculcate and spend tax money on, so as long as mandatorg testing scores are within the margin of error these beliefs have no downside to fund with tax dollars?
I mean if we draw no lines than why teach math or have an SAT at all? Being able to identify and record observable facts are not a critical part of education in some people's estimation it seems.
"These are all confounding factors to a simple and frankly wrong narrative...
Unless you have well designed co-hort studies most of the data just isn't really useful."
Interesting that you claim the data isn't really useful, but then are forceful about a narrative being wrong. This seems like the issue you are complaining about - holding beliefs without facts or contrary to facts.
1. "Religion" is not a monolithic category. It is effectively synonymous with "worldview". The content of a given worldview matters. So, pace modern doctrines of "tolerance", not all worldviews are equal. The notion that they are is simply absurd, as they all vary and differ and make different claims.
2. Doctrine is inevitable. As the Chestertonian quote goes, there are only two kinds of people: those who accept dogma and know it, and those who accept dogma and don't know it. A person cannot function without doctrine. The trouble is that the liberal order has managed to convince us that its doctrines aren't doctrines at all. They're just taken to be "obviously true" or "obviously neutral", which they are not.
3. Since doctrine is inevitable, then all schools at the very least insinuate the doctrines of the governing worldview in the manner things are taught and structured and in the way the school is run. In state schools, these are typically liberal doctrines. That is obvious: why would the state propagate doctrines of a worldview other than the one that it is governed by? It may demonstrate a certain tolerance for other worldviews, but the limits of this tolerance are determined by the doctrines of the governing worldview. This is inescapable, and it is dishonest to deny it.
4. Teaching doctrines is not opposed to knowledge. In fact, if a doctrine is true, then learning it is learning knowledge.
5. Now, among schools we classify as "religious", Catholic schools certainly teach both religious subject matter as well as all the sorts of subject matters you might expect to be taught in a school. We're not talking about some weird American Protestant sect here. Fides et ratio.
Too many lean into the crude caricatures of their imagination.
Great comment. Liberalism (in the broadest possible sense of the word) sets itself up as a dogma-free, 'neutral' worldview, when in fact it is teeming with dogma and substantive ideas about what is good and evil.
Hi, geye1234, its good to be meeting up with you so soon after our previous discussion!
I have to say, however, that it seems slightly tendentious of you to pick out liberalism (albeit in broadest possible sense of the word) in this manner. Is there anything to justify seeing this issue as being specific to, or concentrated within, liberalism? Prior to your comment, liberalism has not been an issue in this discussion.
I had heard rumors about it. The first time I really believed it was when a friend of mine was student teaching and was forced to give no less than 50% if the name was on the paper. He said there was one kid who refused to even write his name to get that free 50% and they gave him an incomplete or something instead. I probably wouldn't have believed it until he told me about all this. He ended up getting his teaching license but never taught due to how bad the schools were policy and behavior-wise. I have other friends who are teachers and I'd say close to half will only teach at private schools for those same reasons.
The key thing is that in the US there are two distinct kinds of religious schools: indoctrination focused, like "evolution bad, women stay in kitchen" level in some cases, focused on "the Bible" and anti-secularism, and these usually have an academic cap because they're not going to do advanced math or stem, or teach accurate history, because of the focus on religion.
On the other hand you have more traditional religious private schools, whether Catholic, segregation academies, etc., and these kinds usually have good academic standards and are probably more comparable to what you refer to. They are also rarely intersecting with the charter system.
So in theory their charters are more secular, but they're going to get as far up on the line as they can.
I have (a not close) relative who is sending his kid to a different charter where their whole website is about how much patriotism and flag rah rah is in the curriculum and how they adhere to traditional gender roles, but for the life of me I can't remember the name.
I also have numerous acquaintances working in various capacities in schools (public, charter, and private) and have to say I regard all the things mentioned in the linked articles as severely under-reported. Retaliation against teachers and staff who try to get kids services at charters is a thing. Public school districts and counties having to foot evaluation costs for parochials is a thing. Refusing and avoiding evaluations for kids in charters so they can be kicked out for behavior and then the publics have to foot the evaluation costs is a thing.
I completely think if local districts (and/or families) had a spine or the right legal resources they could be suing charters who've not even tried to follow or service existing IEPs left and right when those kids are constantly enrolling in publics mid year (with those kids generally exhibiting broad spectrums declines against academic and IEP goals).
I’m baffled by this. My wife worked at a charter school and it was nothing like this. The teachers were constantly trying to get certain kids out of their classroom who were actively threatening staff and other students, and the administration would never do anything. The assumption being that losing a kid would hurt numbers and impact funding.
Your wife needed to know how to demand special ed evaluations most likely (or get the parents to), since that would create costs and cause action. What you're reporting is typical for a profit driven charter that doesn't have much pressure to show high academic performance and is driven by profit motive. Keeping the kids in the seats keeps money coming in. Probably didn't have much of a wait-list to give them more discretion with behavioral bouncing.
Half of her kids had special education needs. You “demand special ed evaluations” and the result is, the kid comes back to your class with an IEP and congratulations, you now have to create an alternate version of each lesson plan.
The parent comment captures the education system’s attitude toward teachers perfectly. The problem is always due to a shortcoming of the teacher, and the solution is always the responsibility of the teacher.
TBH that sounds a lot more similar to the average public than most charters I've heard employees discuss.
Do you have pull out programs, self contained classrooms, etc.? How common is it to have manifestation hearings for the kids your wife thinks are risky in the classroom?
I’m realizing the term “charter school” is probably overloaded.
The charter school my wife taught at (in CA) had to prove itself to the district and fight for public dollars, hence the inability to lose any kids (and dollars).
The charter school I went to years ago sounds more like the kind you’re talking about. Basically a private school that received no public funding.
Huh, I've never heard of a charter model where the local district had any oversight.
To me a charter is just a school that is privately run, usually for either ideological or profit motives, or both, that parents can choose to send their kids to instead of a public school, and which gets money from the state, usually a bit less than a public school would, for each attending child. In turn, the charter school is usually subject to different and less stringent oversight and standards than the public school.
Unlike publics, usually charters do not have to accept mid-term enrollment and parents have to figure out transportation. In demand charters can cap their enrollment and use a lottery for admissions.
Okay, I think that probably describes the charter school my wife taught at, though it was not run for ideological or profit motives, it was inner-city.
The key though is this point:
> gets money from the state, usually a bit less than a public school would, for each attending child
So the charter is under the same pressure as the public school since compensation is directly tied to headcount, the top priority is keeping kids or you won’t survive. I’m not sure how the schools you’re mentioning are able to push kids out and not lose money. Maybe they’re just in affluent areas.
All that to say, I’m not a huge fan of charter schools but I doubt they’re the source of our problem.
One side of the coin is basically softened McCarthyism as a school, which "encourages families to be actively involved in religious organizations of their choice" and [paraphrasing] "supports the national Motto of the United States: In God We Trust."
The other is taking advantage of the angst among professional and white collar parents and runs a curriculum several years ahead of grade level, and takes advantage of filled wait-lists to create high pressure and somewhat rigid environments where kids who don't perform at a high level in even one subject get held back a year and get pressured to leave.
I can't speak to the more "American Exceptionalism" oriented schools as much, despite relative ubiquity - I've heard of them refusing to service IEPs (or resisting doing evals) here and there.
But the BASIS style ones are constantly sending kids back to the public schools that are somewhat psychologicaly damaged despite performing at quite a high level. Typical policies are things like "After grade 9, students must maintain an average score of 3 or above on all AP Exams or the student may not receive full financial support for AP Exams beyond the six that are required for graduation." I have the impression that students go back to publics because of things like not wanting to repeat a year because of struggling with Mandarin and developing self esteem issues. I think they (BASIS) also have non required courses available that have extra fees associated and aren't actually included with the state funded enrollment.
Both angles are able to effectively select student populations that cost less to serve than the student population overall and avoid IEP kids and services for them.
Obviously there are others, but anecdotally, other types don't seem to have a lot of longevity.
Funding varies by locality, but yes, the public schools get funding from the federal government, the state government, and the local government. Most local funding is provided by property taxes, which can vary by local housing valuations. Most state funding programs try to allocate funds to equalize the per student funding across the state to help correct the discrepancy by locality. Federal funds are usually grant related, but oftentimes those grants are need-based or targeted to higher needs groups (% on IEP).
Some of these articles are just generic bashing of school choice, and not relevant to the discussion about whether charter schools reject or push out more challenging students.
e.g. the last article is about Arizona's ESAs, which compete with charter schools.
I'd say the conservatives in AZ are transitioning AZ to ESAs over charters because it lets them fund religious education more easily and with less oversight, does away with the equal access fiction behind charters, and is a way to mess with budgets.
Why is this a bad thing? Its possible I'm missing some nuance because I have not dealt with the US education system in a long time, but I think one fundamental problem with universal education is that one really bad student can easily ruin the education of dozens of others.
If anything, there should be some sort of behavioral/educational reform schools, probably operated in something closer to a military fashion, to help out not only these kids, but the countless others who they'd otherwise be disrupting.
This issue is one (of many) for why if I did decide to have my children go to school in the US it would be at a high performing private school. I'd actually prefer they have the less bubbly experience of public school, but I'm not willing to tolerate having their classes disrupted by kids who are just bee-lining to jail anyhow. I had that upbringing and I think it substantially delayed my 'educational maturity'.
Exactly, ~10-15 years ago Denmark decided to start integrating "problem" children into the regular classroom and scores have dropped across the board. Currently private education enrolment has been massively on the rise since private schools are not forced to accept these kids, and/or they can more easily kick children out.
What's really sad is that now the parents are not only paying extremely high taxes to cover the standard school, but now also are having to shell out extra to make sure their child gets the eduction they deserve.
Why this is shocking is that Denmark (the whole nordics) has an extremely strong public education tradition which is being very quickly eroded. Just 20 years ago private education was seen as something for the elites (e.g. Royalty) but now it's becoming an expected expense.
Almost everyone is frustrated by this: teachers, parents, students, as well as their special needs counterparts. The only people winning are the politicians who get to morally grandstand.
As someone that spend most of school except for the last year in my hometown's low ranking one I cannot compreheend why you would integrate problematic kids like that.
I was doing my best, but with others' constant attention seeking behaviour (shouting, interrupting the teacher, and other more insane things) it was impossible for the teacher to teach and me to concentrate.
If it wasn't for my last year at a private school I wouldn't be where I am now.
I understand they want to create an environment where the kids may feel guilty or something else, but the problem isn't with who they're with in school, most likely. And by problem I mean the CORE problem. That is in most cases a deep issue at home.
In Ontario, Canada they completely got rid of special needs classes recently (so they aren't treated as second class or something), as well as gifted classes, and from two teachers and one child psychologist I talked to it's been a nightmare. Some classes will have 5+ disruptive students constantly yelling or fighting. And in the younger classes when a student has a "meltdown" they aren't allowed to send the kid to the principal, instead they clear the other kids out of the class to wander the halls, bring in the social worker lady (who is always on call), and wait until the problem kid calms down. They told me it was happening at least once a week in one classroom.
We're basically running social experiments on kids.
There are problem kids, but the effect of concentrating them is much worse than the effect of distributing them, much like piling things on one section of net is more likely to break it than distributing them.
There is, in IMO, a ceiling to the percentage of behavior and academic IEP kids you can have in one classroom without it impacting regular kids substantially. (Though for very extreme kids it might just plain not be viable or safe at all.)
Historically reform schools have not been an effective model, but they were also often basically just a way to keep problem kids away from more normal ones and would have treated teen pregnancy the same as severe autism the same as emotional disorders.
You do see more and more self-contained rooms in publics for severe case kiddos that can't be in normal classrooms, which is a bit similar.
Which is to say one child really can ruin the experience of other children. But making it so that private schools and charters have means to keep down the ratio of destructive traumatizers, IEP time wasters, etc., compared to publics doesn't solve anything. It just artificially makes public schools seem worse, cost burdens them, and creates more social stratification.
You basically end up with a tiered system where most public schools have turned into bad reform schools that can't avoid underperforming. Meanwhile, it's not that charters are good, it's just that they've been able to take advantage of two layers of selection to filter out cost centers that decrease performance.
And in turn you are now damaging the prospects of any kid whose parents aren't interested in keeping them out of the public school compared to the other options.
My public school denied education to 99% of students because they let the 1% of students who shouldn't have been there disrupt the classroom every single day.
The outcome for society is that every single student got a sub-par education, because things moved at a glacial pace, had constant interruptions, students felt unsafe, the pace was so slow that students felt bored, etc.
A far better outcome would have been to make a small public school for the highly-academically oriented students (top-10%, future doctors and engineers) a disruption-free normal curriculum (middle-80%) and a special-needs school for all the students who had learning disabilities, behavioral issues, etc which prevented them from being able to thrive in a conventional classroom.
> special-needs school for all the students who had learning disabilities, behavioral issues, etc
What about the students that fit into more than one group? I did not "thrive" in any classroom nor would I still. I had behavioral problems and a learning disability (though I was still an A/B student). I still became a software engineer, but I didn't know I had a learning disability until my 20s. Had I know when I was a kid, where would I have been placed? I had special needs, which were completely ignored (and still are), but I turned out average to above average enough.
Well actually, I did have one instance of being in an 'under the table' special needs program when I was in 6th grade. I was never told why I was in this program for part of every school day, and my parents were never informed. Idk about anyone else, but I did not enjoy being treated like a 'lesser person' because of something I could not control.
Also, what good are these special needs programs for students with learning disabilities when there is no place for many such people in the working world? If anything, I find these programs set people up for more failure than success. The golden rule of non-visible disabilities is to never mention them to any prospective or current employer. Often times, it's a one-way ticket to an eventual PIP/lay-off/firing. 1.5x time on tests is cute, but virtually no employer is giving someone 1.5x on a deadline because they are disabled.
Unfortunately schools err on the side of IEP kids because not enough non-IEP kids' families sue them for being denied FAPE because of the classroom antics of severe children being mainstreamed. (Whereas they are frequently in litigation with insane parents of IEP kids.)
"Any child of any ability, race, socioeconomic background has a right to an education."
Public education is notoriously bad at addressing the situation when some particular child frequently disrupts education of twenty others who would like to make use of their right. This is something that the noble proclamations like yours tend to just ignore.
"The charter school - better for your child? Sounds like it.
Better for society, no."
I'm curious how you think this is true. If it's better for the children in them, then they should have a better education and be more likely to contribute to society yo their maximum potential. So there's the positive side. What we would have to show is that there is a significant negative side for the students still in public schools that was not there before. The only real argument I've heard from this was about how to reduces funding from public schools. However, it also reduces costs for the reduced number of students, so it doesn't seem like a big factor. In fact, most studies show public schools having significantly higher per student budgets than the charters. So what is the negative impact on society?
Don't these scores sort of suggest a negative impact? We've been leaning further and further into charter schools in the US and I don't see where education is thriving.
If there are positive impacts on society, I would think you could very easily show us over the past 20 decades. We have more than 10x the number of charter school students today than in 2000.
==In fact, most studies show public schools having significantly higher per student budgets than the charters.==
Charter schools are generally considered public schools, but generally aren't called "public schools" due to the charter structure. "Public schools" in this example are TPA (traditional public schools). TPAs have higher per student funding than charter schools.
You are presenting a generic correlation between downward trending test scores and prevalence of charter schools. Are there any studies with a real correlation that corrects for other factors? Let's remember that correlation is not causation. Especially consider that achievement was already on a downward trend and that 10x number you quote is only resulting in 6-10% of all public school students today. Even if there is a correlation with the number of charter school students and a decrease in test scores, what is the actual reason or mechanism? Just saying overall scores are still going down and charter enrollments are up doesn't show anything.
It is. They can't put it in bold, but they can avoid special ed evaluations and services, have policies that make undesirable students repeat grades so their family will put them elsewhere, and more. It's just very much "unofficial."
A charter school is a public school that may provide instruction in any combination of grades (transitional kindergarten through grade twelve).
A charter school shall admit all students who wish to attend the school; however, if the number of students exceeds the school's capacity, attendance shall be determined by a public random drawing.
That's the thing with on-paper rights... there is always a way around denying undesirables their rights. Be it onerous voter ID requirements as a tactic of vote suppression, onerous requirements for paperwork and apply systems [1] or kafkaesque processes [2] to make it harder to get unemployment insurance, or in schools looking away at instances of bullying that aren't outright violence, unfair grading and other forms of retaliation.
Magnet and charter schools have a lot in common. You could look at state charter school statutes as encouraging school districts to be proactive about setting up academy model and magnet schools.
No, that doesn’t reflect the situation. Charters enshrine the public school revenue by allocating some of it to “better” schools. They make it harder to lower school district revenue as they (in the eyes of parents) make the established system work better.
That’s different than aiming to reduce district tax levied. I’ve read a variety of statements from charters themselves, including that one. There are also proposals to privatize administration of school districts using the same reasoning. In education, no matter who is speaking, typically the focus is on performance, retaining teachers and meeting more diverse student needs.
I think it just has to be understood that "choice" means choice for some but not for all, and society has to be OK with that. (I'm not OK with it).
In a system with a mixture of public and private schools, the public schools will always be a necessary feature of the selectivity of private schools. Not only to accept "problem" kids, but also to operate in areas where it's not profitable to run private schools, or in cases where private schools run into financial and management problems and must be bailed out.
So it's not so much that the private schools are "better" but that they both function as interdependent parts of a system.
On the other hand, there will always be a need for special education... mainstreaming all kids in one classroom hasn't worked very well either.
2. Religious private schools do not need to meet the same educational requirements as public schools.
3. Charter schools have some selection pressure, but public schools dont have a profit motive.
4. "Charter Schools" as a category is wildly varied, with some excellent institutions, and others that are grifting off the taxpayer dollar.
The most important thing however, is the vicious cycle they introduce. Public schools are required to educate every child, regardless of the expense of doing so. Charter schools face no such requirement. So when charter schools exist in that system, every student that leaves the public school and takes their little chunk of funding with them inherently makes life more difficult for the public school they're leaving. This in turn increases the incentive for every other high achieving student to leave, which makes the public school's situation worse and worse each time.
GP talked about charter schools as being non-secular. Where I live, charter schools are secular. Is that not the case where you live?
Charter schools where I live are required to educate every child. Admissions is via lottery. Is that not the case where you live? (Of course, this doesn't apply to private schools, which definitely cherry pick students, but we're talking about charter schools.)
Re: "public schools don't have a profit motive", this is technically true, but there are plenty of people working for public school districts who contribute little yet draw large salaries and lifetime pensions. A charter school that doesn't make ends meet is forced to shut down. A government-run school district that doesn't make ends meet gets given more money. (This is true in San Francisco at least. I realize that teacher and admin compensation varies widely across the US.)
BTW in most places (even places like Arizona which are relative pro school choice), the per-pupil funding of charter schools is much much less than the per-pupil funding of government-run school districts.
Because charter schools (at least here in California) have to accept any student, just like any other public school, they don't contribute to the cycle you describe. (Obviously private schools do, but I don't think you're advocating outlawing privately-funded schools.)
> Charter schools where I live are required to educate every child. Admissions is via lottery. Is that not the case where you live?
Here they’re required to educate BUT they can do a lot to deter expensive students. If they don’t have support staff, kids with special needs won’t apply. If they require a test to advance grades, those kids will leave. If they require lots of homework, anyone without a nanny or stay-at-home parent and lots of support will leave.
My wife used to teach at one of those schools and it followed an arc over a decade where they started claiming they were cheaper but once they added the legally required support staff they were only cheaper to the extent that they paid their staff less by hiring young teachers and burning them out before they got old enough to expect better pay or have major health insurance expenses.
That is basically the arc of every charter school: every few years you’ll hear about someone doing miracles on a tight budget, but over time it’ll either disappear as regression to the mean sets in or turn out to be some form of selection to avoid expensive children. There just isn’t one weird trick an entire profession has somehow missed.
In my state charters often advertise religious-adjacent or patriotic angles despite being ostensibly secular. Though it is more common for true religious schools to be private non-charters, which sets them up to have some extra hoops for state funding.
In AZ also keep in mind that the funding difference is complicated by education scholarship accounts, which are bankrupting the state.
I agree, and I think a policy shifts should include:
* NOT subsidizing private schools
* NOT determining school funding based on child count
Like so many things, schools are a public commons / good / investment in functioning society that a given goal should be determined, books kept, waste prosecuted (criminally), and services provided according to a defined standard. Then whatever that costs rolls into the next budget as the base tax rate for that year.
Which is entirely not how things are run right now. Government inefficiency frequently relates to 'yearly budgets' and places that look to continue to spend a given year's budget because otherwise it'll go somewhere else. Rather than just doing what's necessary when it's necessary irrespective of how much or little doing the most effective and efficient thing costs.
There's also the elephant in the room on how IEP (btw for the acronym haters: "Individualized educational program". Those of older gens may know it as "Special Education") needs more funding but absolutely no one wants to give that funding to the specialized care-takers/educators who train specifically for such situations. Definitely some not so PR-friendly reasons for this. It's a mess all around.
My daughter has a genetic condition and needed an IEP. The middle school she attended would not give her an evaluation until we indicated we knew they were legally required to evaluate her and they should consider this an official request.
All my previous requests were rebuffed as they tried to steer her towards a 504 plan (504 is not legally binding).
I had to pay to have her evaluated by a Pediatric Neuropsychologist (about $500, despite my excellent American Insurance). He diagnosed ADHD and referred to a Neurologist who found nothing, but referred to an MRI, which found stroke damage, then we were referred to a geneticist who identified her mitochondrial disease and put us into a Pediatric Developmental office.
Finally we got some resources that helped us navigate the school issues to get her IEP.
It was so much time, money, and effort.
One district in my area has lost like 40% of their school psychologists since the pandemic resulting in widespread use of 5/4ths contracts, expanding case loads, uncovered schools, and offers to do extra cases at piece rates. It's (IMO) trending towards some kind of collapse.
As a taxpayer I hate waste. As a long-serving business owner I accept that some level of waste is a cost of doing business.
There are several problems with the concept of "waste".
firstly it's most obvious in hindsight. It's much less obvious in foresight. We spend money with an objective in mind, and in some number of times we fail to meet that objective.
Take marketing as an example. It costs money to attend an event. Will we get a return? sometimes no.
Of course the easiest way to avoid waste is simply not to take risks, not to spend anything. If we prioritize "avoid waste" we create an environment where "nothing is attempted".
Your suggestion to criminalize this with personal accountability makes things worse. What you'll get is no money spent at all, because the risk of spending -any- money could see you in jail.
Secondly, in order to criminalize waste you need to detect it, measure it, prosecute it, defend it, and so on. None of that comes cheap. So you spend money to save money?
Of course we already do this to some extent. We keep books. We have audits. If we detect waste we might examine it, consider alternatives, learn lessons to avoid it. But that's a long long way from the costs of criminal prosecution.
Thirdly you create perverse incentives for bad actors. Don't like a teacher? Accuse them of waste. You can probably find some cause. Your competitor got a contract at a local school? Be sure to loudly proclaim that as waste because you "would have done it cheaper". Sure you would do a crap job that may have failed in 2 years instead of 20, but who's to say?
Send a few teachers to jail - does that increase or decrease the likelihood of people growing up wanting to be teachers?
So yeah, I dislike waste. But preventing waste comes with a price, not just in money. Be careful in what you choose to optimize.
> every student that leaves the public school and takes their little chunk of funding with them
Is this a state thing? In California, I don't get to stop paying taxes if my children go to a charter school. Or, am I just not understanding the point you're making re: "funding?" Are charter schools not also public where you live?
You lose a head at your school, you lose funding for that head. Some heads are more expensive to educate, like special education students, but the state gives you the same amount for each head. So as a school, you might want to keep the cheaper to educate heads and get rid of the more expensive to educate heads.
But if, theoretically, there were a way to count students, and also a hypothetical way to count money, someone might draw a correspondence between them?
Eventually school districts do close schools that have emptied out as kids have gone to charter schools (source: spouse is on school board and they are closing public schools because kids have moved to charter schools)
[0] I strongly disagree with the conclusion that, if government-run schools have excess capacity, then we should prevent charter schools from being opened or expanded.
Any source for this? My understanding is that most states have criteria for what needs to be taught in a primary and secondary school, regardless of public or non-secular. In my experience (attending, touring, friends who are teachers), the non-secular schools had higher standards.
"concentrating behavior issue and IEP kids in public schools"
Public schools in many areas are increasingly doing this by sending G-IEP to entirely separate schools. Indirectly, they are also doing this by increasing school population. Smaller communities tend to better self-police. The biggest issue is the lack of appropriate involvement followed by the lack of leverage the school has. In some areas, some non-public schools even get district IEP resources for their IEP students, so they aren't all avoiding it.
Why would "COVID" only impact the US? It's a global pandemic? If anything the comment would be exclusively about education policies. Because the education policies around COVID would be worse. But also just looking around at my own region of the US, there's a lot of talk about how to lower the school budgets where I live. Which would be lowering on top of inflation's effects Seems like a pretty easy correlation in my head.
Aside: Your 2 comments are sounding alarm bells in my head, and I can't pinpoint why. Especially when the numbers seem to demonstrate the exact opposite of what you're describing in your first comment (scores amongst white students seem to shift downward).
I'd want to hear more about this on a global level, but there's many many accounts on how children in schools became more unrily and apathetic in class post COVID. As well as this rising narrative on treating teachers as "babysitters" more than educators. I haven't heard as such in other countries.
Doesn't mean it hasn't happened elsewhere, but that may be a cultural issue due to many of the social safety nets that exist in other countries simply not being a thing in the US.
There have definitely been problems post covid in the UK - a huge increase in mental health issues for a start, and a definite effect on academic achievement (exams were made easier for a few years afterwards to offset it). IMO it accelerated existing deterioration rather than doing anything new, but it did have an effect.
The government certainly seems teachers as babysitters to some extent. There has been a push for more time in school, from having breakfast at school to more after school activities. More parenting being taken on by schools (I just read about "supervised toothbrushing" in schools on .gov.uk).
How well schools coped with it varied too. My older daughter's school (a well funded sixth form college) had relatively good IT, enough laptops to give them to kids who did not have computers at home, headsets and cameras for teachers etc. My younger daughter was home educated so it was easier to adapt but she still missed out on quite a lot of things, or had to do them online instead of in person and while she came out of it OK, its must have done some harm.
Breakfasts aren’t provided as a form of babysitting. The prevalence of breakfast clubs in schools in low income areas should be a hint as to why they’re instituted. After school activities are more babysitting adjacent, but if parents can’t afford after school minding but really need to hold down a job, it seems like a great idea that also gets kids into sports or chess or whatever.
Why they choose to do it in school rather than providing parents with more money so they can afford to feed their kids is also a hint as to why they are instituted.
"if parents can’t afford after school minding but really need to hold down a job"
i.e. if you want to expand the workforce by providing child minding on the cheap through schools.
The references within and citations of this paper * are as good a place to start as any.
It's Australian and more specifically focused on Victoria, a state with some of the longest global lockdown periods, while more generally comparing to other global educational research on the pandemic effect.
No one paper has all the answers you seek, but a handful of nodes in an interlinked web might do the trick for you.
My oldest is in 4th grade now, and the most relevant maths are from 2nd and 3rd grade. Basic division, multiplication, and fractions matters (3rd grade) - but so does adding and subtracting multi-digit numbers (2nd grade) because multiplication/division is now multiple digit (which is the actual, new, 4th grade material). Interestingly, she used almost no adding and subtracting in 3rd grade, to the point where the teacher supplemented required coursework to help stave off attrition, so you could actually get by 3rd grade while being terrible at adding and subtracting.
Her covid year was kinder. It made her cohort pretty bad at writing. But that seems to have largely worked itself out over the past 4 years.
It's not clear to me when these tests were taken, and that kinda matters - were they by people who were starting 4th grade, finishing 4th grade, or completed 4th grade?
It matters because it tells us what they missed due to COVID - if these 4th graders had 1st grade for COVID, I'm not sure if that would be a huge deal. The most relevant bits are also taught in kinder, and they cover adding and subtracting again in 2nd. But if their main COVID year was 2nd, I could see 4th being a huge problem, especially with the general lack of adding and subtracting in 3rd grade.
> My oldest is in 4th grade now, and the most relevant maths are from 2nd and 3rd grade. Basic division, multiplication, and fractions matters (3rd grade) - but so does adding and subtracting multi-digit numbers (2nd grade) because multiplication/division is now multiple digit (which is the actual, new, 4th grade material)
Are the standards really that low? AIUI in most developed countries w/ the significant exception of the U.S. (and to a lesser extent, other English speaking countries), 4th and 5th grade math are for teaching the earliest elements of pre-algebra. (Meaning that algebraic expressions are very much not used, but the style of reasoning is clearly intended as preparation for that). They may get a lot of practice with fractions or multiple-digit arithmetic, but the basic procedures are not new to them in 4th grade. It seems to me that the U.S. educational system is underserving these kids quite significantly by not catering to their potential and actual skills for mathematical reasoning, which while not fully developed (and very much tied to "concrete" skillsets, as opposed to a real capacity for abstraction) are still quite substantial.
You might double check that the way you number the grades is the same as the way the parent comment numbers them. For example, as I understand it, in the US most kindergarteners are 5 years old and most first graders are 6, while in NZ most first graders are 5 years old and most second graders are six. So to convert from US grades to NZ you add one because in the US kindergarten isn't given a number.
The US has a highly regional system, but as I understand it pre-algebra is taught starting around sixth grade (~11 year olds), which may line up a little closer to your expectations.
> The US has a highly regional system, but as I understand it pre-algebra is taught starting around sixth grade
In most advanced countries this would be the beginning of Junior High a.k.a. Middle School. By that time the students have been thoroughly introduced to pre-algebraic reasoning already, and are broadly getting practice in it (and being taught some more advanced notions around, e.g. exponents and powers, which are of course foundational for later teaching) as preparation for actual algebraic expressions to be introduced.
I can attest that in this US state, 6th graders are being taught those same concepts, at least at the local middle school. (systems of equations with single unknown, introducing multiple unknowns, powers, roots, types of numbers, irrationals, simple geometric problems using multiple shapes, pythagorean triples..)
Anecdotal data from Estonia: I just checked what's the level of maths we have for the 3rd grade (which here corresponds to 9-10 year olds):
(Google Translate):
* reads, writes, sorts and compares natural numbers 0–10,000;
* presents a number as the sum of units, tens, hundreds and thousands;
* reads and writes ordinal numbers;
* adds and subtracts numbers mentally within 100, and in writing within 10,000;
* knows the multiplication table (multiplies and divides by single-digit numbers mentally within 100);
* knows the names of the members and results of four arithmetic operations;
* finds the numerical value of a letter in equations by trial and error or by analogy;
* determines the correct order of operations in an expression (parentheses, multiplication/division, addition/subtraction).
So you have elements of pre-algebra but actual algebra will be covered only in the 4th/5th grade (10-12 yo).
There is a strong push in many districts to delay algebra until high school, since many students are ready for it much earlier but would get an advantage over students who aren’t ready for it. But ya, we are short changing our students while China mostly isn’t (at least in urban schools).
> 4th and 5th grade math are for teaching the earliest elements of pre-algebra
at one large public school district -- 80 elementary schools, middle schools and high schools -- almost half of 8th graders cannot do basic elementary school math problems.. (edit) official estimates are that about 13% of the students do not finish high school at all.. that is the local school district here in a crowded port city in California near San Francisco. also note that more than forty unique non-English languages are spoken at homes existing in this school district.. also a non-zero number of homes where serious drug abuse occurs.. things like that..
Even 40+ years ago algebra was not introduced until 8th grade (13-14 years old), and only the kids who passed a qualifying exam got that option—the rest took it in 9th grade.
My experience in India is that simultaneous linear equations were taught in the 7th standard (12 years of age). I looked it up and it is common https://www.youtube.com/watch?v=beMAypc7ju4
I do not recall it being particularly difficult and most students were able to do this stuff. I'd say that by the end we had 100% success at this out of the 50 students or so in my class. Is this algebra in the US or is it the group theory stuff we studied later on. The group theory stuff was _much_ later (12th standard - 17+ years) and we didn't go too advanced. Mostly simple stuff like proving something is a group or Abelian, etc.
The syllabus I studied was the Tamil Nadu State Board, which is considered less rigorous than the Central Board, so I can only assume the kids elsewhere were studying more advanced stuff. But overall, that sort of timing hasn't hampered me or most of my classmates from then, so one must assume it's not too bad to study group theory that late.
> I do not recall it being particularly difficult and most students were able to do this stuff. I'd say that by the end we had 100% success at this out of the 50 students or so in my class. Is this algebra in the US or is it the group theory stuff we studied later on.
Your parent comment is talking about solving a single linear equation such as "5x + 2 = 1". That's where "algebra" begins in a US pre-university context.
In a university context, "algebra" does indeed refer to group theory, and the basic concept of manipulating a numeric variable goes by the more elevated name "college algebra".
Thank you for explaining. Hard to believe that 13 year olds could fail to do this, or at least formally manipulate the equation till they have a satisfactory answer. Something is wrong with pedagogy or the process of practice.
What's wrong with the pedagogy is the idea that no one should be taught any material until everybody is capable of learning that material. Variable manipulation can be easily learned by 4th graders. But it can't be learned by all 4th graders, so everyone has to wait.
Just in case you think I might be misleading you somehow, here's a cheat sheet product for a "college algebra" course; again, "college algebra" refers to the material that would normally be covered in or before high school, except that it's being covered in college. So the idea of this product is that current college students will buy it to help them understand what's going on in class, or to review for a test.
Many 4th graders would have genuine trouble grokking the notion that a variable (a "letter") may be used in an expression to stand for some arbitrary number. This is why it may be more sensible to reinforce quasi-algebraic reasoning at that age by indirect means, such as practice with non-trivial word problems and with e.g. computing expressions that involve a variety of operations w/ rules of precedence, parentheses etc.
Yeah I'm just saying that even at that time, algebra as a subject was high school or for the "advanced" 8th graders. Calling 4th/5th grade math topics "pre-algebra" would be a real stretch IMO.
I have a preschooler, it's been observed by the local preschools that kids need OT and other interventions at substantially higher rates in the covid cohort than in the non-covid cohort. We are likely to see an ongoing wave of kids experiencing some disruption due to the covid period.
For much of my child's formative infant year, people were screaming bloody murder if we wanted our infant to develop a sense of facial expression in strangers and in public. They were also hell bent on banning much of the social events that people might bring infants to that may help develop early socialization. You do not get a more neuroplastic year than the first one.
There was a lot of stuff going on where one has to wonder how much we disproportionately sacrificed the children for disproportionately the elderly. It still deeply bothers me what effect this could have had on my child.
Only time will vindicate this belief. If you said the same thing two years ago you would have been shouted down and labeled as a “bad person”.
It now is getting to the point where this topic can be discussed without it immediately devolving to attacks.
I agree with your take, humans are also very poor at making long term tradeoffs. Just look at the environment and global warming and what we are doing as a species to stop that.
You also have to consider how covid lockdowns further split class divides. Since we didn’t produce as many things for 3 years there are less things to go around which causes the price of things to go up, hence inflation. Those who belong to unions and have higher leverage (tech workers, MBAs, etc) can push for higher wages to keep up for inflation but those who do not have that leverage are left behind and become poorer.
Im a little bit worried about the reverberations of covid which are causing global instability, and possibly war. A second order effect of lockdowns and inflation is that it makes any government which was in power during lockdowns very unpopular, this has and will continue to lead to more dramatic leadership changes across the board which further reduces stability.
It caused a shitstorm for our family. The prices of houses went 3x in our area due to covid era ~0% interest rate policies, leading me to work so hard to try and keep up I had health issues and was working ~80 hours a week just to be able to buy 1/2 the house I could have before the great leaders printed their way through their policies. I finally managed to secure housing but only after working twice as hard and physically building the house myself one block at a time with completely undeveloped land (this was the only real estate not exploded into oblivion by mortgage bidders), something last done by common people circa the great depression.
The cure may have been worse than the disease. They basically carpet bombed anyone who started a family right at the time the pandemic broke out.
There is general discontent in the population, who would have expected that a CEO being assassinated would be the strongest point of unity for the American people maybe since 9-11.
As more average people get pushed to the margins it will accelerate change. People will realize that it isn’t about left vs right, or race, or creed. It is about class, and those in the higher class echelons don’t want you to know that.
IMO the real culprit there isn't "people screaming bloody murder," it's that because we have no social insurance for pandemic type events, lots of single parent working households, lots of two income working households, lots of one child households, etc., so kids could not get sufficient socialization at or near home despite getting lots of socialization at or near home being totally possible, especially if parents were able to organize a bubble pairing to another household with kids. Instead, kids screwed around on mobile devices at home by themselves while parents worked rather than engage with them.
Many societies covered their faces when outdoors all the time for practical reasons - to protect against the cold, the sun, dust, etc. Some for cultural or religious reasons.
Babies in as diverse of backgrounds as Alaskans to Arabians were raised for thousands of years with significant face-coverings donned before going outside as the norm.
Being worried about "facial expression in strangers in public" because people covered their mouths as a harm to your baby's development is... about 10 steps past absurd. This isn't even making mountain out of a mole hill, this is making a mountain out of a grain of sand.
And of course, compared to the risks of permanently-lowered IQ due to covid brain damage, or permanent lung damage or the various fatigue and other issues millions of people suffered from covid, the risk from the infection to a baby is literally millions of times worse.
This is an interesting strategy you've taken. You're basically setting a trap where you want me to specifically go after Alaskans or Arabians to prove my point.
I'm not taking the racist bait.
I have no reason to think it causes any more or less issues in Alaskans/Arabians a anyone else. The fact that Arabians or Alaskans did or didn't cover in public doesn't sway the outcome one way or another. There are all sorts of cultural factors that compound outcomes, and both mainstream America and Arabs have some that succeed with in spite of, others they succeed because of.
i think the impact of COVID lockdowns and de-SAT-ing would be harder to reverse than to build logical and mathematical reasoning into ChatGPT. The former is political and the latter is technical problem. Our whole industry, great at solving technical problems, is throwing tens of billions today, and it will be hundreds tomorrow, to solve the latter. So the math skills for the majority of the population is probably going the way of the "paper map" skills, etc.
The problem is that mathematics education isn't just about learning times tables.
It's also the primary medium schools use to communicate analytical reasoning and deductive analysis. If you cut math as a target without fundamentally reworking curricular elements, you'll have a ton of graduates who are much worse at negotiating the validity of competing logical arguments.
> It's also the primary medium schools use to communicate analytical reasoning and deductive analysis.
That would be news to History and English teachers among others. You need to learn to make a structured and coherent argument based on evidence in many subjects and the evidence for the existence of transfer learning is so weak it seems unlikely basic Math makes the average or even 75th percentile student better at reasoning. Most people follow a formula at best, and forget that much rapidly after the end of school.
I'd differentiate here between deductive and inductive reasoning. While english and history - through argumentation - expose some deductive reasoning, the vast majority of it is inductive.
>negotiating the validity of competing logical arguments
ChatGPT would soon do that for you better than you do it yourself. That begs the question - what is the core competency of humans? I see so far only the one - the ability to discover, to dig further into unknown. I think though once we get ChatGPT a bit further, such an ability would be pretty easy to add, and it would do much better than humans as it would be able to generate, prune, evaluate, verify hypotheses much faster and at incomparably larger scale/numbers.
> ChatGPT would soon do that for you better than you do it yourself.
And this is based on..? LLMs suffer from all logical fallacies humans do, afaik. It performs extremely poor where there’s no training data, since it’s mostly pattern matching. Sure, some logical reasoning is encoded in the patterns, but clearly not at a sophisticated level. It’s also easily tricked by reusing well-known patterns with variations - ie using a riddle like the wolf-lamb-shepherd with a different premise makes it fall into training data honey pot.
And as for the main argument, to replace literally critical thinking of all things, with a pattern parrot, is the most techno-naive take I’ve heard in a long time. Hot damn.
you're listing today's deficiencies. ChatGPT didn't exist several years ago, and will be a history in several years.
>to replace literally critical thinking of all things
Nobody forces you to replace. The ChatGPT would just be doing it better and faster as it is a pretty simple thing once the toolset gets built up which is happening as we speak.
>it’s mostly pattern matching
it is one component of ChatGPT. The other is the emergent (as a result of simple brute-force looking training) model over which that pattern-matching is performed.
i already do it when it comes to the translation from the languages that i don't know, ie. almost all the human languages. Soon it will be doing other types of thinking better than me too.
For anything weight bearing, I really wouldn't trust it for translation. More than once I've gotten an output that had subtly different meanings or implications than the input. A human translator is going to be significantly more interrogatable here - just as a logical reasoner.
The same way the entire US educational establishment officially groups and tracks students for the purpose of outcome comparison: by self-identified racial classification primarily, and by country of origin for immigrants secondarily. Was it not clear from my original comment, where I talked about white Americans and immigrant students?
It’s not so much unclear as premised on a number of false distinctions that make it unfalsifiable. For example, at what point and under what conditions does a foreign-born child of Polish immigrants that immigrate to the U.S. become white such that they move from one group to the other? How would you correlate however said group membership requirements correspond to test performance? Why does your curiosity about the relevance of Simpson’s paradox disappear when discussing the category of immigrants, a group with more diverse membership than any of your racial categories, and thus more probable to being prone to its effects?
That's what I was thinking given it's America, it just makes no sense in relation to the rest of your comment. If you want to know how COVID effected students, the most natural grouping is by school policies with regard to COVID.
Perhaps IQ is becoming a poorer measurement as we progress? As in, IQ tests might test for more attributes that are less advantageous in modern times. I do not believe students are less intelligent now than previous generations. However, I do believe attention spans have a decreased across the board as a whole (adults too).
I'm probably explaining it poorly, but what if IQ was not a test of just 'intelligence', but perhaps some proxy of intelligence, attention, motivation, and other factors combined? Having taken the WAIS-IV myself, I am not convinced the measurements are all they are cracked up to be.
Think about Homer's Odyssey. The entire epic poem was passed down verbally from memory. If I am not mistaken, Homer thought reading would make people less intelligent because reading would diminish the one's capacity for memory. Does not having the ability to recite the entire Odyssey from memory mean we have become less intelligent, or perhaps, maybe we do not need to memorize the Odyssey because our intellectual capacity is better spent on other endeavors?
COVID is known to cause neurological damage including brain damage. I think we're now seeing the collective impact to IQ due to repeated COVID infections.
Sure but be intellectually honest and acknowledge the vast, vast, vast majority of the learning decline was due to closing schools.
Blaming “covid” for learning loss in a group of people who was hardly at risk of any covid issues at all is… well… doing kids dirty. Which is par for course when it comes to the nonsense society enacted during the first half of this decade.
They closed them then re-opened them in time for one of the biggest waves of asylum/undocumented immigration in recent history, part of which incentivized bringing children (who often need ESL and cultural integration assistance) who then are entitled to attend public schools whether or not the school has been allocated the resources for such influx. Such re-allocation of finite resources to include so many students with extra integrative needs would track well with across the board drops regardless of race.
I can't imagine what it is like to be teaching 29 or 30 students and one pops in after crossing the Darien Gap, possibly witnessing people being raped and swept away by the jungle, then having to go on and go straight into learning arithmetic -- and then realize this is happening at mass scale. "What is 5 - 1?" The number of us remaining after we crossed the last stream.
Visa based immigration pathways, especially the non family linked ones, tend to display this effect less prominently since professional visas bring in people who on average paying higher property and other taxes and have had the resources to prepare their children for school in the US.
While there are certainly specific districts and schools where this is a problem, this effect would not be remotely significant compared to the overall US population. There just aren’t that many new immigrants, regardless of whatever sources you’re reading say.
I always enjoy hearing the dichotomy from various sects of society that simultaneously the immigrants are such drop in the bucket they can be educated for a dozen years at costs that would not be "remotely significant" in measurable change of learning in others, yet simultaneously we're lead to believe deporting them is a monumentally expensive effort that would break our backs.
There is weak evidence - some of the oft-cited studies are just online surveys. If you know of a solid study showing long term, widespread, cognition effects, please share. We know in extreme cases Covid can cause brain injury, but in the mild cases that are the norm for early elementary school kids, there is no reason to believe there is any long term physical effect on the brain.
If covid did that, and as Peter dazic said coronavirus are “driven by their spike protein”, then what exactly were the impacts of messenger rna telling your body to replicate the spike?
>... but these effects are definitely exaggerated by the reweighing of population proportions.
Thanks for sharing the data, but your own data doesn't appear to support that comment.
If I'm reading correctly, the overall drop in US scores was 18 points. The drop in US scores of the two largest populations were 16 points and 17 points respectively. This isn't Sampson's paradox at all. This is exactly what it looks like, each subpopulatuon dropped significantly in score between the pre-covid test and this one.
To call a difference in score of 17 vs 18 (the overall average) an "exaggeration due to reweighting" is quite a misleading stretch of interpretation.
To consider, represent, or cause to appear as larger, more important, or more extreme than is actually the case; overstate.
As you point out, the weighted difference is ~17 which you can clearly see in the data. Increasing this by a full point is quite significant and most people do not even have the intuition that it is even mathematically possible to increase 17 to 18. This is why we teach Simpson's paradox. I also posted the data so people could come to their own conclusion. Just because you use a definition of exaggeration to mean some nonsensically large difference, that does not make my claim misleading.
I would say that it’s quite disturbing that anti-immigrant bigotry, draped in the most threadbare example of pseudo-intellectualism, is the most upvoted comment in this thread, but this unfortunately seems to be the order of the day. Perhaps as a corollary to this study, someone should do a statistical analysis of what has caused this decline in reasoning ability among tech workers.
A quick review of the parent comment would tell you that it never rises to the level of being a hypothesis because it is rooted in category errors and false distinctions that make it neither testable or falsifiable. For example, the appeal to comparison of immigrants as a bloc with shifting American fictions about what constitutes membership in its racial categories. These boundaries are too ill-defined and overlapping with each other to be used in this fashion. What is the process by which one transitions from being an immigrant to one of these groups and how would you observe and control for how any of these (unstable) requirements are correlated to test performance? And of course the same appeal Simpson’s paradox would also have to be applied to the category of immigrant itself before it could ever be considered in this way, since this umbrella contains even more and diverse members than the racial categories with which it is contrasted.
That's nonsense, it's clearly (obviously!) possible that the math score of a country decreases because of immigrants having lower test scores and immigrant share increasing. Which makes it a valid hypothesis. Any problems with definitions are orthogonal to that.
If you believe definitions constituent to a hypothesis are ever orthogonal to it, this would be a mark in the ledger in favor of there being an overall decline in reasoning ability. By this logic, I can simply say “x is a responsible for a decline in y, as evident by the comparable measure of z” without ever having to define any of those variables and always be correct. Does that seem rational or useful to you?
Look, I can understand what you say, and you can understand me, without us defining all or any of the words we use. We don't need a definition of definition. We can know what things mean without having a definition of "knowledge" or "meaning".
That more abstract claim was never what is at dispute. The OP’s claim instead requires clear and consistent definitions of the categories involved in order to be a falsifiable hypothesis, which were not provided. Your defense of this is apparently that because it also exists on the low end of the spectrum of some basic intelligibility that also makes it a hypothesis, which is, again, also incorrect. It’s a supposition that never rises to this level because it lacks these aspects.
This is what you get if you shun uncomfortable discussions, try to shirk finding explanations in problematic cultural practices and promote right wing idiocy by running around with a half collapsed air-castle of comfy half-truths, kept propped up only screaming power of the civilized half of society.
Controversial statement I fully believe in. It is not a decline. Tech workers were always like that. We just liked to not talk about it and call everyone who notice "sjw". Because it felt shameful.
This is where the Goverment should practice what it teaches. I understand that this data is interesting but it is utterly useless. Useless because race isn't as important as the geographical location and economic situation. Race is what you can use once you've narrowed down the demographics to the specific location and socio-economical impact, but it provides little substance without more contextual data.
If a poor neighborhood in a major city is failing math it is the a combination of the neighborhood being poor as opposed to race.
It also has specific undertones... and often people will come with a bad conclusion whether they subconsciously recognize it or not. My guess a better thing to also look at (besides what you pointed out) would be school funding. We have a problem with this in the US. Schools are going massively underfunded and we can observe a massive disparity from school to school. On the political end of this issue, there is a concerted effort to choke out school funding and to push private schools that are funded with tax dollars.
Are they underfunded though? How much is enough? Chicago for example pays probably at least 15k per student yet scores poorly. How much do they need? 20k? 50k?
Fund utility is good in addition as well. Plenty of well funded schools fail. Sometimes it is what those funds are being used for that actually drive down education.
Thanks for the links to the data. I wanted to see if they broke it out by state, but I don't think they did.
I did notice many of the top countries have this fun footnote, "National Defined Population covers less than 90 percent of National Target Population (but at least 74 percent)". Which probably translates to they didn't measure the worst performing kids which likely gooses the numbers pretty decently...
The U.S. has this one, "Met guidelines for sample participation rates only after replacement schools were included." Which means some schools declined to participate so they had to ask others.
I am not a race or genetics expert, but I think "Asian" is not a race.
Asia includes the Indian subcontinent, including India, Pakistan and Bangladesh. Natives of the subcontinent are different from Orientals, such as Chinese, Japanese and Korean, for example, but both are Asian.
Neither is "White". An Australian is completely 180 opposite of Canadian in almost everything and there is almost nothing common except monarchy and English language.
Are Amish white? People from Turkey? Sicilians? Argentinians?
I find the whole concept of "race" perplexing and catering to the lowest common denominator.
You could make the same point about the actual color “white”. You can point to various items of different colors and ask “is this thing actually white?”. You’ll have some people agreeing on most of these things, and disagree on some of these things. Would it make the whole concept of “color” meaningless and useless? Hardly.
If you had some people that were from India, some from Japan, some Siberian Russia, then actually I could see the answers being significantly different
You are missing my point: Asians - vietnamese, chinese, japanese, naga indians, have very little in common from educational/ecnonomical/social point of view.
Same with "whites" (whatever the definition is and for the life of me I just dont understand).
An Australian moving to Canada has a culture shock that is inversely similar to a Japanese going to Paris.
Americans are weirdly still clinging on to the race concept
There are common physical traits that form a family resemblance.
There’s also skull shape and nose length. But what’s the point of binning by these traits? US usual racial division is nonsensical (and I’d say offensive if I was american) and no one dares to question it. You literally blow the question of genetical origin to 100x size compared to the rest of the world. And yet don’t even do it in a way that could make sense instead of being a phenomenon similar to racism but by ignorance and meaningless overgrouping. And then avoid talking about it cause it’s a taboo. It is uniquely American.
>It’s probably the place where discussing race has the least social tolerance.
No. the rest of the world doesnt discuss it because it makes the least sense. I gave you examples above. It doesnt make sense to put an Aussie and Canuck in same bracket, neither does it make sense to put a Thai and Japanese in same bracket for whatever data point you want.
Even within India a sikh has nothing in common with naga has nothing in common with Tamils.
Race is an absurd abstract. Why not hair color? or eye color? or eye shape?
to show what I mean. See those words in the context of the sentences in which they occur, and it can be seen that you are not giving any evidence for your claims either.
Do your own research, if my comment is so important to you.
Also this is HN, a forum, not a court of law. Tons of other users on here, regularly and casually make comments which may seem false to others, without giving evidence for their statements.
>Notice that a charged label on a Wikipedia page is enough for you, a skeptical person, to make an absolute conclusion.
Don't try to mislead, by using words like "charged label". The conclusion is clearly made by the Wikipedia page, not by me. I merely quoted it. Anyone who doubts that can go read it first, before making "absolute conclusions".
>Notice that a charged label on a Wikipedia page is enough for you, a skeptical person, to make an absolute conclusion.
Notice what charged label on the page [1]? By common sense logic of conversation, if you considered that I was using a "charged label" (whatever the heck that means) in my argument, the onus was on you to, at a minimum, mention that label, which you clearly did not do, although I think I can guess which one you mean.
>If it’s so obviously false, can you share the landmark study or experiment that disproved it?
How about you first sharing "the landmark study or experiment" that proves it?
And I wonder if you read the whole article, seeing that the word "discredited" (referring to phrenology) appears at least 5 or 6 times in the article, in many cases with citations.
>All that’s true, but you can still identify them trivially. There are common physical traits that form a family resemblance.
That's exactly what I implied by my above comment ( https://news.ycombinator.com/item?id=42383753 ), which means the same as this part of yours: (i.e. you can still identify them trivially. There are common physical traits that form a family resemblance.).
But mine was referring to the differences between Indians on the one hand, and East Asians (and some others) on the other hand.
Here, the ”East Asians” includes Chinese, Taiwanese, Japanese and Koreans, among others, and the "some others" also includes Tibetans, Nepalese, Bhutanese and Sikkimese, and Burmese, and people of the North Eastern states of India, such as Assam, Meghalaya, Manipur, Nagaland, and Tripura.
All of these above people can easily be distinguished from (the rest of) Indians by their narrow eyes.
I don't know what is so difficult to understand about this. Nor is it discriminatory. It is simply stating obvious facts, that anyone can visually ascertain for themselves.
It is the people who behave in a discriminatory way towards anyone, based on where they come from, or their ethnicity, who are the real racists, not those who talk in an innocuous way about distinguishing visual characteristics of various categories of people.
>Yeah, I’m not buying that race is a uniquely American construction. It’s probably the place where discussing race has the least social tolerance.
(
>American
Say the word right, dude. USian, not American. :)
)
Discussing race may have the least social tolerance in USA (or not), but when it comes to actual behaviour, i.e. racism?
And I do not mean to single out the USA. There is plenty of racism in Europe and Asia too. I have myself experienced it in both places, and have heard of plenty of anecdotes about it from people whom I trust.
There's probably high genetic kinship (Britain) for Canada and Australia amongst 'white' people, the US too has a large body of genetic kinship with Britain, it's definitely not your 180.0.
Yes, that's one of the main findings from the article.
> The extent of the decline seems to be driven by the lowest performing students losing more ground, a worrying trend that predates the pandemic.
> Scores for the highest performing American fourth graders were about the same as in 2019, but the lowest performing students — those in the bottom 10% — saw their scores drop by 37 points in math and by 22 points in science compared with similar students in 2019. The lowest performing eighth graders saw their scores drop by 19 points in math. One in five U.S. eighth graders scored below the low benchmark, meaning they lacked even basic proficiency.
> This gap between high- and low-performing students started to widen before the pandemic for reasons that are unclear. Since then, other research into post-pandemic academic performance has found widening gaps across race and income, even as many middle and higher income students are doing well.
It's pretty obvious: our society, because it's based on representative government, requires a baseline of general welfare sufficient that as many citizens as possible have leisure time and are not subject to PTSD-inducing levels of stress. This is at odds with the increasing inequality and increasing cultural and class stratification within our society. As it is increasingly only possible for upper-middle class and up parents to care for children effectively, the rest of the child-age population will continue to be worse and worse off, while the upper middle class pull the ladders up after themselves out of concern and fear over the future welfare of their own children being completely zero sum.
The most urgent things that need to happen to stop this are:
- Ensuring sick leave, vacation time, and improving wages, especially on the lower end, such that 1.5 incomes are sufficient for 2 child homes so that parents can have time for their children.
- Policies that severely disincentivize vouchers, private, and charter education so that there are fewer incentives to try and create "tides that selectively lift boats."
You seem to be confusing "Wanting that stuff" and "Believing that solution can be implemented by the same people I do"
Americans, overwhelmingly, agree that things are not going in the right direction.
What they are split on, is how to fix it, and what is the greatest driver.
No, I'm not at all. If you really think the GOP is going to promote legislation to "[Ensure] sick leave, vacation time, and [improve] wages, especially on the lower end", then you are seriously delusional. You'd have to be a complete idiot to think that GOP politicians are going to push for any of this stuff, because they've never done so in the past, and it certainly isn't part of their current platform. Those goes even more for disincentivizing private school, vouchers, etc., since those are things the GOP has been actively promoting for ages. They've also said nothing about wanting to increase housing supply.
About #2 (disincentivize charter education): parents created charter schools because they disagreed on the choices that was made for them in public education. It is very hard to fight this, as 1- they see a huge difference in test results 2- they want to give their children the best chances in life.
Finland solution was to ban all private education. Once rich kids were forced to go to public schools, then suddenly education became a larger part of the gov budget, those parents spent more time asking for reforms, volunteering etc.
They have one of the best performant school systems in the planet. And all it takes is not allowing a percentage of the population think of education as optional
> How does the existence of private schools enables this thinking?
Here is a simple example. There is a family with a combined 6k income, their expenses are 2k on rent and 1.5k on child private school, grocieries etc account for 500, savings another 500 and the remaining 1.5k goes on taxes.
Now the goverment proposes a 10% tax raise to improve public schools. Do you think this family would vote for or agaiinst that bill?
Now lets propose a counter example. Same salary, same expenses minus the private school, which gives the family 2k of savings per month. Now due to shortages in public schools they have volunteered at their kids school, helped with field trips, met other parents and families in their neighbourhood. Now the goverment does a 20% tax raise for public schooling. Would this family be more or less willing than the other family to vote yes?
By simply being involved you and your vote become resposinble for it. If you can pay your way away, specially if you stretch your finances to afford it, then investing in the system goes against your own interests regardless of the social externalities of your position.
We could have a society were parents of privately educated kids voted for a public school improvement, but psicologically and economically we know they by and large dont. Therefore the existance of private schooling comes at the expense of a fair, well resourced, and functional public education
Those who care about education, can leave to a private school instead, and not spend time trying to improve the public schools, just leave them to their fate: noisy school-is-boring kids sabotaging the classes for everyone.
Or that's how I interpret what GP said. Happening a bit where I live: richer families don't want their kids go to school in problematic suburbs.
> and not spend time trying to improve the public schools
> noisy school-is-boring kids sabotaging the classes for everyone.
It seems that those parents realize that if the system does not care about ensuring that kids in school actually learn things, then they better leave than trying to fix the kids of others (i.e., parents who does not care).
I would argue that it is a net positive for those public schools: those parents still pay local taxes while making sure that there are fewer students -> better student/teacher ratio.
Maybe depends on how hard or easy it is to improve the schools, on how much influence the parents can have? In Finland, apparently it was doable, but that's, in a way, a very different place than here.
No, I'm proposing the opposite: that states try to act more like Norway/Finland by themselves, instead of hoping the federal government will do so (which isn't going to happen).
And yes, schools are funded by local taxes, but it doesn't have to be that way: if states really wanted to, they could take that power away from municipalities. The constitution gives the states broad powers to run themselves as they like.
> There was also a strong correlation between socioeconomic status and test scores. Students from higher income households and those who attended schools with more affluent students had higher scores.
The question is what was the delta within that group (and within other groups). Given we know that those of higher socioeconomic status do better than average, did they improve compared to their past selves, stay roughly the same, or degrade despite still maintaining above average results?
If, for example, they degraded, then it would indicate there is a problem happening that even the higher socioeconomic status doesn't protect from despite all its benefits (well it might protect relatively better that other SES, but not enough to prevent the downward trend).
If, instead, they improved relative to their past selves, it would indicate the problem causing the overall trend is found entirely within other SES groups (well not entirely, as it is possible the impact still hit this group but that other factors more than made up for it, but to keep it simple, we can ignore such a possibility unless we see even better data that can parse out these more complex relationships).
The Simpson’s paradox part is factual, and has been known for over a decade courtesy Steve Sailer, who is an immigration restrictionist. A fairer critique of the poster is that they’re unoriginal (rather than that they’re blaming immigrants, which I didn’t see in this post).
> While US has excellent education, and immigrant children in America do better than their counterparts in their countries of origin, they don’t do as well as the modal group of white Americans, so as the composition changes...
it doesn't say the immigrants are to blame, it just strongly implies that this could be a likely reason, furthermore for some reason to be an immigrant is to not be white which certainly was not the case when I immigrated to America.
A more plausible interpretation is that the education system is failing immigrant families currently. This has the benefit of both matching the evidence and not having racist implications.
> courtesy Steve Sailer, who is an immigration restrictionist
That doesn't make this argument credible. Is there any credible source?
> rather than that they’re blaming immigrants, which I didn’t see in this post
We know well that the talking points of racism are formed to create ambiguity and use dog whistles. That goes back half a century and probably much longer. These are the talking points.
I didn’t suggest this makes the argument credible. In fact, I fully expect that it should make one more skeptical of the argument, which is part of the reason I put that in there.
> While US has excellent education, and immigrant children in America do better than their counterparts in their countries of origin, they don’t do as well as the modal group of white Americans
My experience does not track with this at all, especially in the context of the very wide brush you're applying it. I was involved in a lot of extracurricular events around math/science in school, and it was not the white kids winning those events. There were some pretty racist comments about it as well which makes it stand out even more in my memory.
Comment is true of immigrants as a group, but it’s a remarkably heterogeneous pool. When you break immigrants down down into East/South Asians versus White/Jewish versus others, the differences in educational outcomes between these splits is large.
I would go a bit further and say that Asians should be in their own group altogether and not compared to anyone else until the other groups are remediated. They are so far off the scale, especially on the high end, that I'm not sure comparisons help until we can get everyone up to, at least, their neighborhood.
It's strange to split whites into those two categories since Jews represent a trivially small group, why not Italian/Greek and German/Scandinavian ancestry that is somewhat similar to East/South Asian split?
This is idiotic. I lumped Whites and Jews into one category (since there is a subset of Jews that don’t consider themselves white), East and South Asians into another. Read more carefully before you cast aspersions.
I think a lot of racists (or race realists, depending on your perspective) are today a lot more concerned about the influx of southern natives than about the blacks. The black proportion of the population has been steady for a long, long time. The disparities there are a big issue, and people on all sides are concerned about it for different reasons, but I wouldn’t be surprised if GP is actually referencing immigrants in this case.
>I think a lot of racists (or race realists, depending on your perspective) are today a lot more concerned about the influx of southern natives than about the blacks.
Don't forget, American Black people speak American English (albeit frequently with a dialect, but they're capable of code-switching to standard American English). New immigrants from south of the border do not. It would make sense for many people, even if racist, to consider Blacks a part of their "tribe" since they share a language and culture, to an extent, whereas Spanish speakers from the south would be seen as a different tribe.
But I agree, blacks and Pacific Islanders combined don't comprise enough of the population to be causing the drop in numbers. I removed the lowest performers from the data entirely, and could only count one group that actually performed on a level indicating a readiness to process next concepts. And that group, I'm assuming, is comprised of a large number of immigrants.
So, until it can be explained why no one other than asians performed satisfactorily, I'm just not sure the data supports this drop being about immigrants. We obviously have a lot of work to do fixing the education system in this country, and the people out here talking about immigrants or blacks are, for whatever reason, trying to get in the way of doing that work.
The data is clearly stating that there is a problem here. And, in the US anyway, that problem exists in every demographic but one.
ETA: Again, the level for readiness to process next concepts is set at 550 for TIMSS.
For anecdotal flavor I’ll mention that I briefly worked in Hollywood and casting requests trended towards “BIPOC” rather than “non-white”, including in roles that had no evident racial component (i.e., the request was indicative of an apparent desire to have a nominally “diverse” cast without any real storytelling commitment to the racial diversity of said character(s) and there was a pronounced bias in these characters (constructed to be "diverse") towards a particular sliver of racial diversity).
I could be misunderstanding HN User dylan604, but I suspect the kids winning those competitions were not white or black. I could be wrong, but I think we all know who we suspect were winning those competitions. And it wasn't white or black kids.
That is exactly my point. Our parent commenter was disputing the fact that the broad stripe of “immigrant” populations are responsible for our reduced and over all disappointing scores. My comment is meant to insinuate that the overly broad (and thereby meaningless) redirect to a vaguely monolithic “immigrant” population (one invariably implied to “not yet be assimilated!”) is exactly that - a redirect from
the genuine problem group in this country (an unnervingly underperforming one), a group which is not comprised of immigrants.
Evidently “immigrants” is a popular term of convenience for all folks – left, right, and center.
Maybe I'm reading the data wrong, but it seems to say that Pacific Islanders are the lowest performers, blacks the second lowest, and Hispanics the third lowest? Again, if I'm incorrect, please do correct me.
So I add up, say, Pacific Islander and Black population percentages, and it's maybe 16.5%. I mean, the weights... well put it this way, again, I could be wrong, but when I remove them from the data, I can see only one group that meets the target mark for being ready to learn more advanced concepts.
ETA:
- Misread charts initially and mistakenly said that two groups met the mark. Nope. Only one group met the mark. Point being, even if we drop Pacific Islanders and blacks from the numbers, the US wouldn't even make the top ten. We should be, at least in the top five in my opinion, but that's not happening right now. And that problem is occurring very much across the board. Again, someone correct me if I'm wrong.
- Also, not sure if it is well known or not, but the target TIMSS mark is 550 for being able to process next concepts.
Which is another way of saying that there's nothing being done about inequality, the problem is worsening. Hard to teach poor kids, hard for kids to get out of poverty when the government literally removes anti-poverty measures because it wants an underclass of workers to do shitty jobs for shit pay or alternatively populations it considers reserve labor (to keep wages down for employed people) or expendable.
Is there any data to back up your claim that immigrant kids are worse at math in US schools?
It doesn't match my personal experience. It is _well known_ in Europe that the American K-12 education system is weak.
Anecdata: All the exchange students from my middle- and high-school (in a third world country in Europe) came back saying they already knew the math that was being taught in the US school.
Well known and incorrect. If you control PISA scores for demographics, the American education system is fine to great. You can see American Whites outperform most other countries. American Black's outperform parts of Europe, including Greece.
Why would we not? If I didn't control for demographics, I probably wouldn't have my kids in our local public schools (in the wealthiest county <? - Santa Clara> in the country) because the Great Schools ratings show only mid-pack achievement. If I do control for demographics, though, I see the white & Asian kids are doing fine and it's the rest who are struggling, primarily for a combination of socioeconomic & immigration reasons. Given this, I'm comfortable sending my kids to our neighborhood schools because I'm in the fortunate demographic where kids will do well no matter where you put them.
It's the same for any educational achievement measures, including PISA. Kids from households with two parents who are working professionals and college educated will do fine most of the time, and kids without those privileges will struggle most of the time.
It kinda makes sense when looking at local schools to send your children to. It makes no sense when comparing countries in a major international study, that's just massaging data to make yourself look good
Every country has immigrants though, you can't just selectively remove them in one country and not the others, that is not an apples to apples comparison.
Because it's a random axis to split the data on and you only do it for the US, which makes zero sense to do when comparing entire countries.
What about Tunisian immigrants in France? Maybe they'd be at the very top? What about Whites in all countries? Maybe White kids across the globe are better than White kids in the US, which would put the US at the bottom?
We cannot conclude that "US education is amongst the top in the world" from the chart you presented.
> The education system is the same for all the kids
This is incorrect, certainly an ideal we'd all like but far from reality. The educational experiences and outcomes of 1st and 2nd generation immigrants can and often does differ significantly from native-born students. Seeing the difference in the data/scores should clue you into this and helps us understand the socioeconomic impact on student achievement that immigration has. Language is often a large factor where 1st and 2nd generation students may be speaking a different language at home than they are in school. The US has significantly more immigrants than other countries in the world which is why not controlling for it skews the data disproportionately.
If that table is to believed, the US is kind of middle of the pack if you sort by percentages, at 15%. It might even be middle of the pack for developed countries, too
I've counted OECD countries[1] (roughly 40 developed countries), and the US is in 15th place, on par with Spain. Hardly worth writing home about :-)
[1] This is the fairest comparison because people immigrate to rich countries in general. The only poor countries with lots of immigrants are generally safe poor countries right next to war-torn countries, which get lots of refugees.
Its especially pertinent here that a lot of the countries that outperform the US have significantly higher percentages of immigrants, yet they don't seem to be failing them in the manner that the US education system is:
Switzerland: 30%
Sweden: 20%
Canada: 21%
Germany, Austria etc.
Given how the US does schooling where zipcode matters as much as it does, something tells me you can get from A to B a lot quicker just by controlling for school district wealth or something
Conjecture, but it may be to focus on the native born population as a better metric for education quality since they have (presumably) been a product of that system for their entire life. Contrast that to a 1st gen immigrant who a major amount of time in a different country's system; testing them after a short stint in the US tells us much less about the US educational system. It's harder for me to think of a reason why 2nd gen should be removed, unless the assumption is the educational attainment/integration of 1st gen parents heavily biases the results of their kids. I don't know if all that holds up under scrutiny, though.
Also, because demographics aren't distributed evenly geographically, I think there is probably a case that the education system is different for different races (to the extent that racial geographic distribution is different).
2nd gen are removed so that there are fewer poor people with uneducated parents who can't help them at home because the school system is bad in the sample.
That's what I was alluding to, but it's unclear how sound of a methodology that is. It seems like a better approach would be to control for parent educational attainment directly, rather than use some imperfectly correlated metric like immigration status.
How you select immigrants would dramatically affect these results, so it makes sense to exclude them. A lot of East and South Asian 2nd gen achievement is due to them being the children of highly skilled immigrants - they’d thrive in almost any setup. If USA imported only PhDs from Latin America, their kids would be, on average, stellar achievers. For this reason, it makes sense to eliminate them as a group, in the interest of simplicity.
The money is taken by middlemen. Spending lot on education is necessary if you want good results, but it's not enough if there is lot of corruption in the system.
We spend more on education that practically anyone else, and we spend more on it that we had in the past, with very little difference between now and then.
Some of the countries, like Japan, are close enough to racially homogenous that the results wouldn't be affected. For France or something it would be more interesting to see racial breakdowns, if the data existed. I don't think that means the comparison in the chart is arbitrary
That data does exist but is mysteriously excluded. Perhaps because it doesn't support the race bait? Moreover, contrary to the figure's claims, PISA does not exclude immigrants. It's just a hodgepodge of crappy statistics. Don't get fooled by anonymous Reddit posters.
> and immigrant children in America do better than their counterparts in their countries of origin, they don’t do as well as the modal group of white Americans
East and South Asian immigrants probably beg to differ.
Of course you’re right that children of US Asian immigrants are ahead even of US whites, I mention this in my different comment in this thread. However, since there are few of them relative to other groups, they don’t affect the distribution much, so I didn’t mention this for the sake of clarity, as it didn’t detract from my main point.
There are not few of them relative to other groups. In fact there are more of them than several, more native groups, like Pacific Islanders, Native Americans, and Mixed Race people
Well no, East and South Asian immigrants to the United States do tend to outperform the median in their country of origin. Acutely, in the case of South Asians.
So that's half of the statement, the other half is that those two groups are not a large fraction of the total immigrant population. As such, they affect the conclusion, in the sense that removing them from consideration would make the difference in question starker, but they do not change it, because it's a statement about a whole of which they are but a part.
> they don’t do as well as the modal group of white Americans, so as the composition changes, and the whites become less of a statistically dominating factor
How do poor whites compare to the rest of white people, as well as other races in their socio-economic strata? I suspect test scores corelate more strongly with household income and wealth than with race. However, in America, income/wealth and race are also correlated, which may make one an inadvertent proxy for the other.
> • Blacks from families with incomes of more than $100,000 had a mean SAT score that was 85 points below the mean score for whites from all income levels, 139 points below the mean score of whites from families at the same income level, and 10 points below the average score of white students from families whose income was less than $10,000
I'm confused. Your first sentence was how do our whites compare to others and this comment addresses. You suggest the driving factor is socioeconomic class, which the child post directly addresses.
I asked a very specific question, and I fail to see what's confusing about my first sentence. I asked how poor whites compared to other non-poor white people, as well as poor people of other races (i.e. controlling for income). Your reply provides a link comparing how non-poor black people compare to white people across all income levels, leaving my question unanswered.
If I were at a Toyota dealerships and I asked you "How does this Corolla hatchback compare to other Toyota models on the lot, as well as other brands in the same 'compact car' category?", and your response is that the Nissan Pathfinder (a mid-size SUV) has worse gas milage than all Toyota models. That doesn't answer my question at all, because non-Toyota SUVs were never under consideration.
Ah I should have stopped reading after your first sentence, because the next sentence suggests the broader point you're getting at is that difference in test scores is attributable to mostly economic factors. Here is your second sentence:
> I suspect test scores corelate more strongly with household income and wealth than with race.
The question of whether people in higher socio-economic class score higher in standardized tests is obviously true and uninteresting. The real question is if the correlation trumps other factors like race (which you hinted at by your next sentence). Kind of like asking "is this SUV safe for a Toyota". Obviously its safer than smaller Toyota vehicles, the pertinent question is whether its safe compared to other SUVs.
So yes, students in higher economics. You can google 'socio-economic test scores'. Here's a top result:
> Socioeconomic status (SES) and SAT scores are positively correlated: Students from higher income backgrounds generally achieve higher scores, and “21.2% of variance in SAT scores is shared with SES, as measured here as a composite of mother’s education, father’s education, and parental income.” The researchers note that the “source of the SAT-SES relationship is likely due to some combination of educational opportunity, school quality, peer effects and other social factors.”
The US has absolutely abysmal primary education while spending eye watering amounts of money on it. This is plainly obvious to anyone who has ever lived abroad with kids.
Just say that you think white people are special, it doesn't matter how they actually do, and Asian immigrants don't count except when comparing between non-white groups.
I am Asian, born in the US, and it's definitely the wealth aspect. You can only end up here in the US legally as an immigrant from Asia if you've completed some type of advance degree (or are going to do that in the US) or are extremely wealthy.
So the cream of the crop of Asia is competing against everyone of the native born population and you then obviously get this kind of disparity here.
Asians love to talk about how we focus more on education, our culture is better, blah blah. But go back to India or China and you will find lots and lots of people who don't value education and don't have much care for discipline or culture or frankly any sort of society. There's a reason India has had a Trump-like leader for multiple years and it's not because of a focus on education.
And congrats to them and their families. The stats still show that Asians who move to the US are overwhelmingly likely to be moving here to go to college, to go to graduate school, to continue their established skilled career, or with already significant assets. All of which make them usually in the top 10% (if not 1%) of their respective countries. Asians just cannot get visas to move here to work in restaurants. The only ones who can are ones who come here as refugees or in other such ways which make then the minority of Asian immigrants to the US
You have you differentiate within immigrants too. East Asian and Indians do significantly better than white Americans. White Americans are usually a distant third.
By what measure? As far as I know it is poor, in absolute terms and relative to its peers.
For the rest, do you have any evidence for the statements, or that the outcome depends on race (and not, for example, income, local school quality, etc)? A very large portion of the US population is neither immigrant nor white. And blaming the immigrants for undermining 'white' people is also part of a hateful disinformation campaign, so I think we should be very careful with statements that might be used to spread that hate.
Edit: Even if immigration status were correlated, that doesn't mean that immigration causes people to be worse at mathematics. It could be, for example, that immigrants suffer disrcimination and a lack of resources - like some other groups that have lower scores.
At the very top, the US excels at math. We consistently place at the top or near the top in the IMO for instance (https://maa.org/news/usa-first-at-imo/). Yes the team is largely children of immigrants, but they are Americans, too.
Did you assess every other school system "at its very top" and compare? This seems to be very clearly a double standard. Consider that IMO numbers are going to be biased due to the US's larger population compared to other highly developed countries.
It's ironic that your comment includes immigrants to show that America is great, while other comments exclude immigrants to show that America is not so bad.
Aren't those prodigies who get identified and given special education that is not available to most? They may not be all the useful for judging the quality of math education in general in the US.
Yes, but the ability to identify prodigies and give them the resources to realize their full potential is part of an education system, too. I don't know the full details of the IMO participants, but the team is geographically diverse.
Of course, another aspect of the education system are the resources given to the average student, and I don't think there is much debate that the US could do better here.
From 2022 stats:
Roughly 4 million students in each grade in the US.
8th,9th,10th,11th & 12th graders = 20 million.
So 20 million students are eligible for AMC10/AMC12
Of them, only 300K took the AMC10/AMC12
Of them, only 10K were invited to take the AIME
Of them, only 500 were invited to take the USAMO
Of them, only 50 went to the MOPs
Of them, only 6 went to the IMO & won.
The article is about 4th & 8th graders, of which we have 8 million.
Now, suppose those 4th graders get to 8th grade, then the 8th graders would be in 12. That makes both cohorts eligible for AMC10/12. At that point, they become 8 million out of the 20 million. So 120K of them take the AMCs, 4K take the AIME, 200 get into USAMO, 20 get invited into the MOPs, and best case 2 kids make it into the IMO.
So from this grand experiment, 2 kids will emerge the victor, and we are supposed to back-extrapolate that 8 million kids do ok. Aaalright then.../s
US has excellent education by the standard, commonly used international measures. In PISA, for example, white American children score better than any European country, and are only outscored by a handful of East Asian countries. These are all in turn outscored by US Asians, putting US on top yet again. US has a bunch of lower-performing population groups, who bring the overall average down, but even these perform better than their foreign counterparts. US blacks perform better than blacks in any African country, immigrants from South America perform better than kids in the countries they came from etc.
> For the rest, do you have any evidence for the statements, or that the outcome depends on race
Not sure what you’re asking for. Did you not know that there are large differences in means of test scores between various ethnic and racial groups in US? This is a commonly known fact, I don’t think really require me to provide specific evidence.
> And blaming the immigrants for undermining 'white' people is also part of a hateful disinformation campaign, so I think we should be very careful with statements that might be used to spread that hate.
Not sure what you’re talking about, but I get the feeling that you’re trying to smear me with allusions and innuendos.
The pre-college US education is not excellent. I know because I went to a US public high school. Here are some of my observations:
1. The standards are low. Here are some examples:
- My high school calculus class went at half the speed of a college calculus class. Some of the students still complained it went too fast!
- I took Spanish for 3 years. Native Spanish speakers laugh at my Spanish because it is extremely poor (poor pronunciation and I cannot read a Spanish newspaper).
2. Students and parents are more concerned with grades than with learning.
3. I know of teachers who have been punished because they had high standards. I know of one English teacher who was made a librarian because he failed half of his class. He was the best English teacher in the school, and he had high standards. I know he was good because I had him.
4. When I went to high school, I was surprised at how many students could not write a coherent argument in English.
5. My physics and chemistry classes in high school looked nothing like the classes in college. Basically, they were relatively easy, and I did not learn a lot. In the case of physics, the textbook also did a poor job of explaining concepts.
My main point is my experience with US public education was poor. Classes were easy, students were not pushed, and concepts were not taught. I don't think you can call a school which cannot teach physics, chemistry, or Spanish a good school. I also do not think my experience was unique.
I knew that you were going to get attacked because someone would take your statement as hateful. Unfortunately, it takes longer to collect data than it does to write a dumb comment. They took your factual statement of Simpson's paradox being able to drive one global summary (lower overall scores - a bad thing) counter to the local summaries (increased scores by all groups - a good thing) explained by shifting population proportions as you "blaming immigrants" for the bad outcome when the logical extension of your statement is "hold on, we may have a good thing going on". Ignore @mmooss.
Have to admit the immigration comment also triggered my "blaming immigrants" radar. After a careful reading it was an honest attempt for more information or clarification on how the details might be better interpreted. Something people should do more of when statistics are used as evidence. Actually a very good question.
> US has excellent education by the standard, commonly used international measures. In PISA, for example, white American children score better than any European country, and are only outscored by a handful of East Asian countries. These are all in turn outscored by US Asians, putting US on top yet again. US has a bunch of lower-performing population groups, who bring the overall average down, but even these perform better than their foreign counterparts. US blacks perform better than blacks in any African country, immigrants from South America perform better than kids in the countries they came from etc.
Wow. I‘m really impressed with your skill to interpret the data and build a narrative from it. However, let’s face it, it is just a convenient narrative with a faulty logic.
You cannot conclude that USA has excellent educational system if it delivers the great outcomes for certain advantaged subgroups. It’s logically incorrect and not how efficiency of education system is defined.
It is also wrong to compare black Americans and Africans and generally use non-scientific racial divisions which do not exist elsewhere in the world. Most of black Americans are no more immigrants than white people. They are simply Americans and should be compared to Europe the same way as whites.
> You cannot conclude that USA has excellent educational system if it delivers the great outcomes for certain advantaged subgroups.
To the contrary, you could totally conclude from this that US has excellent educational system at least with respect to these, as you call them, “advantaged” groups. At best you could argue that the system is not so excellent for other, less well performing groups. However, these less well performing groups still perform better in US system than matched groups outside the US, so it is indeed the case that for both higher and lower performing groups, US educational system beats all other systems around the world.
> However, these less well performing groups still perform better in US system than matched groups outside the US
If you pick random data points you can support with them any theory. Comparing white Americans (a random group of people defined by a subjective criteria, rather than by anything scientific) to a whole European country is random.
Now, speaking of the choice of subgroups. If you select best performing subgroups to see which country can support the best talent, why are you not looking at the results of international olympiads?
IOI - China, Russia, then USA.
IPhO - China dominates the list of winners.
IMO - USA has some wins, but China won more.
> Comparing white Americans (a random group of people defined by a subjective criteria, rather than by anything scientific) to a whole European country is random.
“White Americans” is very much not a “random group defined by subjective criteria”. Self-identification as white is extremely highly correlated with objective, measurable metrics like percent of European ancestry, and this also makes US whites directly comparable to population of European countries, which as it happens still are overwhelmingly comprised by people of European ancestry.
I am not selecting best performing group for the sake of comparison, I’m just comparing various natural and obvious population clusters that have been understood and distinguished by everyone completely unrelatedly to the discussion of educational outcomes. The category of white Americans has not been invented to show how great US education is. I am totally interested in comparing educational outcomes of white Americans with other major ethnic or ancestral populations in other countries, it’s just white Americans come out ahead almost every single time.
Self-identification on the basis of race is not objective. Races do not exist, it is a scientific fact. There are many other more reasonable ways to cluster American population. Ancestral angle is important, but only from cultural proximity perspective if you talk about people who are 5-10th generation Americans. Black Americans that are descendants of slaves may have higher proximity to Europe than to Africa, so they should be included in the group that you compare to Europe and anyway that comparison must be based on some theory, otherwise it’s just cherry-picking for building a convenient narrative.
> Self-identification on the basis of race is not objective.
It correlates extremely closely with objective measures like percentage of genetic ancestry from a given continent or historical population, so close in fact that for statistical purposes, it is justified to regard these two as virtually identical.
> Races do not exist, it is a scientific fact.
I’ll happily use different term like “ancestral group” if you like it more, but the (scientific) truth of the matter is that it’s just splitting hairs. I know that in recent years activists worked hard to obfuscate and misinform people on these topics, so I will happily accommodate you.
> It correlates extremely closely with objective measures like percentage of genetic ancestry from a given continent
It doesn’t make this clustering less arbitrary.
> I’ll happily use different term like “ancestral group” if you like it more
I do not care what term do you use for it as long as I do not see any scientific argument for having it. Politics and whatever activism have nothing to do with it.
Races do not exist in the same sense that the periodic table does not exist. Both are constructs over reality, and they are both informative (i.e. science).
Periodic table uses objective criteria for categorization. American race classification is rooted in debunked theories and mostly meaningless today. One can say at least that there exists black subculture, black dialect of English etc among descendants of slaves. How much of that is related to 1st and 2nd generation immigrants from Africa, which have much stronger cultural links to their motherland, speak different languages and may even have different faith? Asian bucket is absolutely non-sensical — there’s either cultural proximity to America or to native Asian cultures, which are very different, so the people are very different. It is very hard to understand why Indians and Chinese should classify themselves the same way. This classification is imposed on them. How is this nonsense informative? It’s just some racist legacy.
In Europe we do not have that system and we don’t miss it.
You cannot be serious. It is very well established by science that biological races do not exist. They remain in the mostly American conversation as sociocultural constructs.
Maybe you at least read Wikipedia to educate yourself?
This is just obfuscation. The population clusters will still exist even if we don't use the word "race" to describe them, and they can be described in biological terms that will overwhelmingly overlap with the sociocultural construct.
You continue repeating this without any scientific evidence, yet any modern source points that racial theories are not supported by genetic research. There‘s no such overlap.
It turns out that if you cluster people together by their genetic information, the clusters that form are pretty similar to the racial groupings that ordinary Americans would understand. See this chart: https://en.wikipedia.org/wiki/Human_genetic_clustering#/medi...
The 4 clusters in the K=4 column are pretty much just Black vs White vs (east) Asian vs Amerindian
First of all, you quote an old study with a very small dataset. We already know by now, that before going out of Africa at least two major genetic branches developed in addition to early Eurasians. At K=4 and clustering by genetic distance, they would likely represent two clusters while the rest of Africa would fall into one of remaining two. Not even close to what average American would understand.
Second, even if in the study you use, it’s „east Asian“. Where you would put Indians? How about southeast Asian people with black skin?
Why this clustering is even necessary when the rest of the world is doing fine without it?
> We already know by now, that before going out of Africa at least two major genetic branches developed in addition to early Eurasians.
Nice of you to acknowledge genetics.
> At K=4 and clustering by genetic distance, they would likely represent two clusters while the rest of Africa would fall into one of remaining two.
I suppose they would.
> Not even close to what average American would understand.
The average American would understand if those two branches lived among us today. The value of science is to inform us of what is occurring now and to predict what will occur next, including the impact of immigration on test scores.
> The average American would understand if those two branches lived among us today
Shall I break the news or you find out yourself?.. ok, I will do it.
Those two branches do live among Americans. Afro-American people have the biggest genetic diversity in America and their genetic subgroups are so distinctive that they require separate testing in clinical studies. I bet you won’t be able to tell the difference between them from their appearance though. Now good luck redefining the concept of race with this knowledge.
If you were not referring to extinct branches I don't know why you thought "they would likely represent two clusters" if they did not already in the clustering given.
> Afro-American people have the biggest genetic diversity in America and their genetic subgroups are so distinctive that they require separate testing in clinical studies.
Yes, and?
> I bet you won’t be able to tell the difference between them from their appearance though.
Maybe, and?
> Now good luck redefining the concept of race with this knowledge.
Good luck trying to deny biological race when you've just listed more evidence for it.
>I don't know why you thought "they would likely represent two clusters" if they did not already in the clustering given.
Read the study with the clustering. I did it, so you should too.
This is my last reply to you. If you need more answers, there’s already more than enough facts for you here to verify and learn something new in the process.
I‘m talking about cultural proximity. The tragedy of American slavery is that it erased any links of slaves to Africa, so modern black Americans have barely any relationship to Africa, still carrying the pain but losing any cultural or ancestral connections. They were raised and educated in a culture that is mostly a product of Europe. It is wrong thus to compare them to African countries since it gives false impression that they are doing great.
> The tragedy of American slavery is that it erased any links of slaves to Africa, so modern black Americans have barely any relationship to Africa, still carrying the pain but losing any cultural or ancestral connections.
Ancestral connection in the quoted sentence means knowing your ancestors, not genetic lineage. It must have been clear from this thread that I don’t deny genetics.
I am talking about the leading group, whatever that may be, why would you split only the United States by a certain dimension and compare it to the entirety of other populations?
Ah, I get it now. Yes, it would be more apples-to-apples to, for example, compare white Americans with ancestral Europeans, by disaggregating scores in European countries by ethnic/ancestral group of origin. However, this would not affect comparison greatly, because in European countries, the children of ancestral Europeans comprise 80-90% of the total, compared to <45% in US, so taking the whole aggregate instead of this 80-90% doesn’t change that much. Same is true for East Asian countries: immigrant population in Japan or Taiwan is pretty negligible, as these countries are fairly homogenous. Finally, for countries with highly heterogeneous populations, like eg India or Indonesia, none of the large ethnic groups is ahead of US whites.
I think I've found the answer and it's simpler than that. There is no analysis splitting whites or other ethnic groups because PISA has national variations of questionnaires (source on the US one below), and in all likelihood no other country splits by race.
It's still weird, probably lacking rigor and methodological soundness, to split American races out and compare them to the bulk of other populations.
It might not be super sound methodologically to only do it for US in PISA, but I don’t think that it affect results significantly. Other high performing countries have highly homogenous populations, so the aggregate score of the entire country is very close to the score of the top performing group. On the other hand, in countries with heterogenous populations like India, Indonesia, Afghanistan or Nigeria, even the top groups are not performing very well. If you know of a country other than US, where the top performing group does significantly better than country average, and their performance is on the level of, say, European average, I’d be very curious to learn about it!
The main methodological problem is how one can leave out like 40% of the United States, and say that the US comes on top (which it still doesn't!). Like, I'm not even questioning the racial split data, but selecting the two groups and saying that the US is the best is a weird flex that wouldn't pass muster in Stats 101.
I’m not leaving out anyone. What I’m saying is that each American group individually comes out out of top, relative to matching groups elsewhere, and the fact that overall we don’t, is just an example of Simpson’s paradox. American blacks and Hispanics are ahead of foreign blacks and Hispanics too, it’s not just whites and Asians.
There are regularly PISA reports on European countries that split the data by natives (n-generation Turks are usually counted here) and immigrants. Sometimes all the immigrants are in one single group, sometimes there is fine-grained data (Somalis do very poorly, probably to nobody's surprise).
I'm with you (just to be clear): the educational system is perfectly fine in the US. The problem is that the educational system in the US is designed to be fine for the subset of kids who come from families that 1) speak English, 2) are educated themselves, 3) hold full time employment.
Whenever these debates about the US education system arise it's important to clarify that when people who complain about the system do so, they're really complaining that the lack of a social safety net is keeping the doors open in public schools for kids whose families (and themselves) don't see value in formal education, can't maintain daily attendance for various reasons, may not have internet access at home, don't have food safety, may have health and developmental issues, and may not even have one parent caring for them.
I don't care whether science shows a differentiation in IQ between racial groups, because that is so much less important than addressing how well a kid will be able to learn when they are being raised in poverty by a single parent without a college education who doesn't speak English natively and may not even be in the country legally. That is the problem.
> The problem is that the educational system in the US is designed to be fine for the subset of kids who come from families that 1) speak English, 2) are educated themselves, 3) hold full time employment.
I think that, to the contrary, the US educational system goes to great lengths to accommodate students for whom English is a second language. Second, I can scarcely think of a way the US education is designed for the children of educated parents. Obviously they do better than children of uneducated parents, but it is hard to even imagine the system where this would not be true; certainly it’s not true anywhere else in the world. Finally, I think that regarding the full time employment of the parents, literally the opposite is true: it’s designed with the opposite assumption. For example, elementary school is not 10 hours long, as it would have been if it assumed that both parents are occupied for 8 hours every day.
> To the contrary, you could totally conclude from this that US has excellent educational system at least with respect to these, as you call them, “advantaged” groups
Wouldn't the educational system being geared towards these groups be an advantage granted to those groups?
> You cannot conclude that USA has excellent educational system if it delivers the great outcomes for certain advantaged subgroups.
This is not what he did. He just explained to you that the outcomes for all those subgroups are basically better in the US than elsewhere.
Even that is a moot point though, because the aggregate US score is already perfectly comparable to e.g. EU level. Even if you cherry-pick EU nations for GDP/capita (Switzerland, Netherlands, Denmark), you can see that they are pretty much exactly on US level (with the EU leaning a bit more toward math).
This also tracks with anecdata I have from family (central europe) that did a year at a US highschool (east/west) and both described the math curriculum as "joke".
To reiterate: Describing the US education system as "bad" (=> compared to peer-nations) is just objectively wrong.
> This is not what he did. He just explained to you that the outcomes for all those subgroups are basically better in the US than elsewhere.
He literally said that USA has excellent educational system. The explanation with statistical gerrymandering and random comparisons followed.
>Even that is a moot point though, because the aggregate US score is already perfectly comparable to e.g. EU level
EU level isn’t great. Besides, EU recently experienced major influx of immigrants (on the scale of millions). USA is #18 in the latest rating, 7-15% difference in the score with top 5%.
And yes, American math curriculum is indeed a joke.
> Describing the US education system as "bad" (=> compared to peer-nations) is just objectively wrong.
I didn’t say it is bad. You are replying to my comment where I basically say that you cannot say it’s excellent based on THAT data.
>You cannot conclude that USA has excellent educational system if it delivers the great outcomes for certain advantaged subgroups.
If group B has characteristics that make education more difficult relative to group A (e.g. lower mean IQ), then if there are two countries with identical educations systems, the one with more students of group B will end up with worse educational outcomes, just because group B are harder to educate regardless of the system.
Second, other than racism, I don't see why we would distinguish between the 'race' of the Americans. The US average is the average. People are no more or less American based on their skin color.
Unless you make the racist claim that math skills biologically depend on race, there is no reason to compare some Americans with people in Africa because they have similar skin color - it's absurd.
> Not sure what you’re talking about
I don't believe that you are unaware of the hate and discrimination against immigrants and minorities, justified by these same and similar arguments. If you said that was not your intent - well, your intent or not, it has the same effect. But to say you don't know is not credible.
Fortunately for you, if you don't want to use race to distinguish you can instead use these characteristics to define your subgroups:
1) kids with two married parents at home
2) kids of parents with a college or professional degree
3) kids of parents who work white collar salaried jobs
4) kids of parents who are native English speakers
If you check all those boxes you're going to see that the resulting subgroup does very well on all standardized tests, including PISA. It will also be predominantly white/European-American & Asian/Asian-American. Those four attributes make the biggest difference in educational attainment, and brown & black families are the ones who are being left behind.
For the next thought exercise, consider what can be done to address this shortcoming in our political, social & education systems.
Sorry, what do you mean by “evidence”? I clearly said that I am talking about PISA results, and described what these look like. You can look these up yourself if you are curious about more details.
> Second, other than racism, I don't see why we would distinguish between the 'race' of the Americans.
For better or worse, this opinion is not shared by the mainstream American culture and policy-making circles. We take great efforts to distinguish between these groups in the context of educational outcomes. It is not surprising, because these different groups very much exist in very objective sense, and quite objectively have different educational outcomes, which makes it useful to distinguish between them for the purposes like the original question, which was whether the educational quality went down recently. My original point that in order to answer this question, you have to distinguish between these groups so that you don’t fall victim to composition fallacy, is largely orthogonal to any discussion about the causes of these disparities, because it still stands regardless of whether the causes are 100% biological or 100% cultural or 100% result of systemic discrimination or whatever.
> Unless you make the racist claim that math skills biologically depend on race, there is no reason to compare some Americans with people in Africa because they have similar skin color - it's absurd.
You are creating a really weird straw man, because I don’t think that even extreme KKK-style racists have much of an issue with skin color per se. Who are you arguing against here? I am extremely confused.
> I don't believe that you are unaware of the hate and discrimination against immigrants and minorities, justified by these same and similar arguments.
Ah, here comes smears and innuendos that by open discussion of clear, objective facts that are relevant to policy making, I’m causing some kind of nebulous harm to some unnamed people via some proxies. Just stop it, I don’t care, and nobody cares anymore either.
While IQ may vary by racial groups, overall math attainment not at the very top couple of percent has far more to do with socioeconomic circumstances & parental education than it does any inherent potential.
This is almost certainly an SES effect; the evidence for biological causation is weak and has gotten drastically weaker with each successive GWAS study. Either way: it's a race war point, and those are unwelcome on HN.
I think the opposite is true. There is large gap between educational outcomes of white and black Americans even if you match them on SES, and every successive GWAS increases the amount of variance in outcomes explained by variants, despite the input data only allowing for extremely crude metrics like years of completed education instead of actual test scores.
Literally the opposite thing, to what you just said, is true: GWAS studies have slashed heritability estimates (which beg the question of biological causation, but also establish a ceiling to it), by something like 80%. To say that this wasn't the outcome biological determinists expected from GWAS would be an understatement.
There is clearly a cohort effect, of students we identify as Black, with markedly lower educational success. Again: that's almost certainly an SES effect. Which shouldn't surprise anybody, given all the other SES effects that apply to that same cohort of people.
No, you are completely misunderstanding what GWASes show. The previous estimate of heritability of things like intelligence are entirely unaffected by GWAS results. That existing GWASes explain small percentage of variance is the problem with the GWASes, not with the preexisting, giant, extremely well replicated heritability literature.
Assume for the sake of argument that intelligence is 100% heritable, and is a result of additive heritability of thousands of variants. Pre-GWAS tools would in that case find something like 0.9 heritability (due to attenuation because of measurement error). Now imagine you run a GWAS that includes only 1% of SNPs of the entire genome, and your intelligence proxy is “has any university diploma”, instead of more accurate measures like eg IQ. In that scenario, you would never expect the GWASes to get even close to actual heritability.
This is where we are today: our genome coverage is relatively low (especially around rare variants), while the metrics we have are crude. Of course we are not even close to heritability estimates! If someone told you that low explained variance of GWASes is consequential for classical heritability estimates, they were either confused themselves, or deliberately tried to misinform you (extremely common problem in the field, with a long tradition from people like Lewontin and Gould back in the day, to people like Gusev today).
> Again: that's almost certainly an SES effect.
You just repeat it this without in any way substantiating it, despite me being very clear that the effect does not disappear even when you match by SES.
No, that's not all the recent studies do; they also attempt to separate inherited environment, and estimate direct vs. indirect heritability.
It's funny, I can point to comments from IQ-fixated people on this site from 10 years ago saying that GWAS was going to settle heritability stuff. All that's gone now, of course. Now the work is apparently fatally flawed.
shortly later
(I'm sorry I'm giving clipped answers; I am also trying to unclog a Bosch dishwasher, and commenting in between draining cycles).
Please, tell me which GWAS study is claiming to estimate entire heritabilty. I am not aware of any. This is because their methodology simply does not allow it. They by design are going to miss some heritability, and the methodology is not powerful enough to be able to tell us how much they are missing. If you don’t understand this, I think you are very confused about what these studies actually say.
> It's funny, I can point to comments from IQ-fixated people on this site from 10 years ago saying that GWAS was going to settle heritability stuff.
Please, do, and bring receipts! You should refresh your memory as to where the whole debate was 10 years ago. At the time, many people still hopelessly argued that intelligence have nothing to do with genes, and the “IQ-fixated” people replied that GWASes will conclusively show that they do. Now that they have, the goalpost has shifted, and people who argued that intelligence has nothing to do with genes are, similarly to you, arguing that it doesn’t have as much to do as “IQ-fixated” people have claimed. Of course, in 10 years, the goalpost will move again.
I haven't spaced on this, but I'm going to take a beat to generate a more rigorous reply. In the meantime: I think the evidence, both from twin studies and from genomics, for genetic determination of intelligence is very weak, has gotten weaker, and describes an effect too small to be relevant to the gaps we're discussing in this story; bringing racial IQ science into a discussion like this is arson.
I'll do my best to back those points up tomorrow.
It's a little weird that you called out Gusev, above; on this topic Gusev, though a geneticist himself, seems more like a popularizer of current research than someone going out on a limb with his own. But check his references: the scientists whose studies he cites seem to be saying the same thing he is.
I was waiting for the reply, but I realized that you might not be able to send one or edit your comment anymore, so here is one from me to give you opportunity to do so. I’m really curious what you found about GWASes or the state of debate around them 10 years ago.
How is this "race war"? No one is saying people should be punished or are inferior. We're talking about why schools perform the way they do in the US, not some "war".
It looks like he's mostly concerned with flame wars and slurs, which makes sense since they destroy conversations driven by curiosity and turn them into bitter fights. That's not at all what is going on here.
The fourth comment down, and this is sorted by date not relevance, is almost isomorphic to this thread. I believe you didn't intend to start a "race war" (that's Dan's term, not mine), but the big subtext of HN's norms is being responsible not just for your intentions but for the effect your comment has on the thread.
It is not at all established that there are meaningful, coherent IQ differences between "races", and the process of hashing that out on this thread will be an attractive (and tedious) nuisance that will attract awful comments --- for instance, like one suggesting "brain size" differences between races are explanatory.
The OP pointed out a highly informative and important point and the majority of the negative responses to it have been from people who want to bash the American education system.
They are using cries of racism and bringing in irrelevant topics like IQ intentionally so that they can shut down legitimate conversation because they want to continue to bash the American education system.
It's not a highly informative point; it's a deeply contentious one that revolves around a narrow(ing) and disputed finding that is drastically smaller than the educational gap we're discussing.
> In PISA, for example, white American children score better than any European country,
European countries also have better scores if we disregard the immigrants. Well, mostly certain immigrant groups. Others do fine.
And those groups that do bad do bad everywhere. Those that do well do well everywhere. It's almost as if something else matters more than the local educational systems.
The source he cites is himself, other than the PISA data.
I cannot examine the entire dataset right now, but from reading the questionnaires I cannot find a single question on race. The only resembling question is on the birthplace of the parents [1], but you cannot infer the race from that, can you? How would you separate blacks then, most of whom would probably have American parents?
Edit: The US version of the questionnaire does ask for comprehensive data on race. I still cannot make sense of some of the elements of the Crémieux source, for example they show the US average (not split by race) in a position which is not what I see in the official data. But it should be entirely possible to analyze the data comprehensively by race.
I have known Cremieux for a long time, and trust him a great deal when it comes to handling and presenting data. In any case, I don’t know what standards you’d find satisfactory. He made the graph out of data he listed on the graph. Anyone can go and verity its accuracy. It being materially wrong would be devastating to his reputation. Thus, if you still don’t trust it, you should just do your own leg work to verify it, instead of asking others, whom you probably don’t trust any more than you trust Cremieux, to do it. Anything more would be unusual and unreasonable, even formal academic peer review does not involve verifying that the graphs are accurately representing underlying data.
How insightful could this possibly be if socioeconomic variables are not controlled for? Clearly the socioeconomic makeup of "US Asians" is not the same as, say, the entire population of Korean students.
> Not sure what you’re talking about, but I get the feeling that you’re trying to smear me with allusions and innuendos.
To be fair:
>> Americans, so as the composition changes, and the whites become less of a statistically dominating factor, the scores are expected to go down, even as the education quality improves.
Reads a lot like "Our scores are going to go down because there are more brown people, even when we try so hard to help them.", and without further explanation, suggests a predicate of "brown people are inherently dumber". I understand that you're more likely suggesting that the immigrants are coming from a place with lower educational quality, which means they have a weaker educational background. Particularly though its a litte odd when the only called out groups are "white people" and "immigrants" with no mention of other groups like non-white natural born citizens (not to mention confusing when trying to consider white immigrants). It has a lot of the elements you'd find in dog-whistles:
* the immediate framing of results in terms of race/ethnicity
* setting up a whites vs the undifferentiated others.
* a declaration that white people are propping up the others and describing whites as dominant. (I know you said statistically dominant, but the word dominant has a lot of connotations no matter how it's prefixed).
* the use of flat, statistical wording around a seemingly odd juxtaposition (the first point).
Doubly so on the internet where it's extremely common to find the person who said the original to be spewing overt racism a bit down-thread.
I don't know if you are racist, and I don't think you intended racism in your comment. I'm just pointing out that it's not surprising someone read bigotry into it.